Pharm CBA 3 complete

Lakukan tugas rumah & ujian kamu dengan baik sekarang menggunakan Quizwiz!

19. Mineralocorticoids include: A. Beclomethasone (several formulations) B. Fluticasone propionate C. Hydrocortisone (several formulations) D. Cortisone E. Methylprednisolone F. Prednisone G. Fludrocortisone acetate H. Aminoglutethimide

G

36. Adverse effects for adrenal drugs of this system include: - Increased intraocular pressure, glaucoma A. Cardiovascular B. CNS C. Endocrine D. GI E. Integumentary F. Musculoskeletal G. Ocular H. Other

G

Interactions: Thyroid drugs

-enhance activity of oral anticoagulants -taking concurrently with digitalis glycosides may decrease serum digitalis glycoside levels -cholestyramine binds to thyroid hormone in GI tract-may reduce absorption of both drugs -diabetic patients may require increased dosages of hypoglycemic drugs -used with epinephrine in patients with coronary disease may induce coronary insufficiency -insulin: decreased efficacy of insulin resulting in increased blood glucose levels -estrogen: reduced thyroid drug activity -digoxin: decreased digoxin effectiveness -phenytoin and fosphenytoin: reduced levothyroxine effectiveness -phenobarbital: reduced levothyroxine effectiveness

Patient teaching

-thyroid replacement drugs best taken 1/2-1 hour before breakfast on an empty stomach to enhance absorption orally, maintain constant hormone levels, and help prevent insomnia -do not abruptly discontinue, lifelong therapy is usually the norm -importance of keeping follow-up visits to monitor thyroid hormone levels, CBC, and liver function studies -brands of thyroid replacement drugs cannot be interchanged -all thyroid tablets must be protected from light -Antithyroid drugs taken better with meals or a snack, given at the same time every day to maintain consistent blood levels of drug -consult with physician before taking any OTC drugs -avoid eating foods high in iodine with antithyroid drugs

The order reads: "Give ocreotide (Sandostatin) 50 mcg subcut twice a day." The medication is available in an injectable form of 0.05 mg/mL. How many milliliters will the nurse draw up for the ordered dose?

1 mL

1. A patient has been admitted for an exacerbation of chronic obstructive pulmonary disease and will be receiving methylprednisolone (Solu-Medrol) 30 mg intravenously every 6 hours. The medication is available in 40-mg/mL vials. Identify how many milliliters will the nurse draw up for this dose. _______

ANS: 0.75 mL DIF: COGNITIVE LEVEL: Applying (Application) REF: N/A TOP: NURSING PROCESS: Implementation

Thyroid Drugs: Levothyroxine (Levoxyl, Levothroid, Synthroid, others), T4

100% T4 (thyroxine) makes its effects most predicable, half-life long enough that it only needs to be admin once a day, switching between brands during treatment can destabilize the course of treatment, thyroid function test need to be monitored when switching products

2. A patient will be receiving oral theophylline (Theo-Dur), 600 mg/day, in three divided doses. Identify how many milligrams will the patient receive per dose. _______

ANS: 200 mg DIF: COGNITIVE LEVEL: Applying (Application) REF: N/A TOP: NURSING PROCESS: Implementation

1. A 10-year-old child will be receiving docusate sodium (Colace), 120 mg/day PO, divided into 3 doses. Identify how many milligrams will the child receive per dose. _______

ANS: 40 mg DIF: COGNITIVE LEVEL: Applying (Application) REF: N/A TOP: NURSING PROCESS: Implementation

NCLEX EXAMINATION REVIEW QUESTIONS 7. The order reads: "Give cimetidine (Tagamet) 300 mg in 100 mL normal saline IVPB tid and at bedtime. Infuse over 30 minutes." The infusion pump can only be programmed to deliver over 60 minutes (mL per hour). The nurse will set the pump to deliver how many mL/hour for each IVPB dose?

200 mL/hr

13. Over secretion of the adrenocortical hormones leads to: A. Cushing's syndrome B. Addison's disease

A

21. MOA for adrenal drugs in general: A. Exert effect by modifying enzyme activity B. Inhibit inflammatory and immune responses

A

24. Glucocorticoids are administered by inhalation for: A. Control of steroid-responsive bronchospastic states B. Rhinitis and to prevent the recurrence of polyps after surgical removal C. Inflammations of the eye, ear, and skin D. Shock, status asthmaticus and spinal cord injury

A

30. Adverse effects for adrenal drugs of this system include: - Heart failure, cardiac edema, hypertension—all caused by electrolyte imbalances (hypokalemia, hypernatremia) A. Cardiovascular B. CNS C. Endocrine D. GI E. Integumentary F. Musculoskeletal G. Ocular H. Other

A

4. When assessing for potential adverse effects of fludrocortisone (Florinef), the nurse monitors for signs and symptoms of which condition? A. Hypokalemia B. Hypovolemia C. Hyponatremia D. Hypercalcemia

A

6. The nurse should question a prescription for aminoglutethimide (Cytadren) in a patient with which condition? A. Addison's disease B. Adrenal malignancy C. Cushing's syndrome D. Metastatic breast cancer

A

7. To prevent the development of oral candidiasis when using corticosteroid inhalers, which instruction is MOST important for the nurse to teach the patient? A. Rinse the mouth after each use. B. Minimize use of an inhaler to every other day. C. Swish and swallow with Mycostatin after each use. D. Report any gingival irritation to the health care provider.

A

ANS: C Excessive use of sodium bicarbonate may lead to systemic alkalosis. The other options are incorrect.

A 75-year-old woman comes into the clinic with complaints of muscle twitching, nausea, and headache. She tells the nurse that she has been taking sodium bicarbonate 5 or 6 times a day for the past 3 weeks. The nurse will assess for which potential problem that may occur with overuse of sodium bicarbonate? a.Constipation b.Metabolic acidosis c.Metabolic alkalosis d.Excessive gastric mucus

ANS: A Stress-related mucosal damage is an important issue for critically ill patients. Stress ulcer prophylaxis (or therapy to prevent severe gastrointestinal [GI] damage) is undertaken in almost every critically ill patient in an intensive care unit and for many patients on general medical surgical units. Procedures performed commonly in critically ill patients, such as passing nasogastric tubes, placing patients on ventilators, and others, predispose patients to bleeding of the GI tract. Guidelines suggest that all such patients receive either a histamine receptor-blocking drug or a proton pump inhibitor. The other options are incorrect.

A patient in the intensive care unit has a nasogastric tube and is also receiving a proton pump inhibitor (PPI). The nurse recognizes that the purpose of the PPI is which effect? a.Prevent stress ulcers b.Reduce bacteria levels in the stomach c.Reduce gastric gas formation (flatulence) d.Promote gastric motility

ANS: A Calcium antacids are not used as frequently as other antacids because their use may lead to the development of kidney stones; they also cause increased gastric acid production. The other options are incorrect.

A patient is asking advice about which over-the-counter antacid is considered the most safe to use for heartburn. The nurse explains that calcium antacids are not used as frequently as other antacids because a.their use may result in kidney stones. b.they cause decreased gastric acid production. c.they cause severe diarrhea. d.their use may result in fluid retention and edema.

ANS: D Simethicone alters the elasticity of mucus-coated bubbles, causing them to break, and is an over-the-counter antiflatulent. The other options are incorrect.

A patient is complaining of excessive and painful gas. The nurse checks the patient's medication orders and prepares to administer which drug for this problem? a.famotidine (Pepcid) b.aluminum hydroxide and magnesium hydroxide (Maalox or Mylanta) c.calcium carbonate (Tums) d.simethicone (Mylicon)

ANS: B Aluminum-based antacids have a constipating effect as well as an acid-neutralizing capacity. The other options are incorrect.

A patient is receiving an aluminum-containing antacid. The nurse will inform the patient to watch for which possible adverse effect? a.Diarrhea b.Constipation c.Nausea d.Abdominal cramping

ANS: D Omeprazole needs to be taken before meals, and an entire capsule must be taken whole, not crushed, chewed, opened, or dissolved in liquid when treating GERD. This medication is used on a long-term basis to maintain healing.

A patient is taking omeprazole (Prilosec) for the treatment of gastroesophageal reflux disease (GERD). The nurse will include which statement in the teaching plan about this medication? a."Take this medication once a day after breakfast." b."You will be on this medication for only 2 weeks for treatment of the reflux disease." c."The medication may be dissolved in a liquid for better absorption." d."The entire capsule must be taken whole, not crushed, chewed, or opened."

ANS: B Smoking may impair the absorption of H2 antagonists. The other factors are correct interventions for this medication.

A patient who has been taking cimetidine (Tagamet) for hyperacidity calls the clinic to say that the medication has not been effective. The nurse reviews his history and notes that which factor may be influencing the effectiveness of this drug? a.He takes the cimetidine with meals. b.He smokes two packs of cigarettes a day. c.He drinks a glass of water with each dose. d.He takes an antacid 3 hours after the cimetidine dose.

ANS: A The antibiotic clarithromycin is active against H. pylori and is used in combination with omeprazole to eradicate the bacteria. First-line therapy against H. pylori includes a 10- to 14-day course of a proton pump inhibitor such as omeprazole plus the antibiotics clarithromycin and either amoxicillin or metronidazole, or a combination of a proton pump inhibitor, bismuth subsalicylate, and the antibiotics tetracycline and metronidazole. Many different combinations are used.

A patient will be taking a 2-week course of combination therapy with omeprazole (Prilosec) and another drug for a peptic ulcer caused by Helicobacter pylori. The nurse expects a drug from which class to be ordered with the omeprazole? a.An antibiotic b.A nonsteroidal antiinflammatory drug c.An antacid d.An antiemetic

11. The adrenal medulla secretes the following catecholamines: A. Epinephrine B. Norepinephrine C. Glucocorticoids D. Mineralocorticoids (primarily aldosterone)

A, B

18. Glucocorticoids include: A. Beclomethasone (several formulations) B. Fluticasone propionate C. Hydrocortisone (several formulations) D. Cortisone E. Methylprednisolone F. Prednisone G. Fludrocortisone acetate H. Aminoglutethimide

A, B, C, D, E, F

Which instruction should be included in the teaching for a patient for whom fluticasone propionate [Flovent] MDI has been ordered? A. "Gargle after using your inhaler." B. "Take the medication immediately at the onset of an attack." C. "Take your albuterol first, followed by the Flovent 5 minutes later." D. "Make sure you monitor your fingerstick blood glucose level each morning."

A. "Gargle after using your inhaler." Patients should be taught to gargle after using inhaled glucocorticoids, such as fluticasone propionate, to minimize dysphonia and oropharyngeal candidiasis. The medication is not used to abort an acute attack. Short-acting bronchodilators, such as albuterol, should be administered 5 minutes before inhaled glucocorticoids to enhance delivery to the bronchial tree. Oral glucocorticoids pose a risk of hyperglycemia.

What would the nurse assess when monitoring for the therapeutic effectiveness of vasopressin? A. Fluid balance B. Patient's pain scale C. Serum albumin levels D. Adrenocorticotropic hormone (ACTH) levels

A. Fluid balance Vasopressin causes decreased water excretion in the renal tubule, thus decreasing urine output. It is used to treat diabetes insipidus, which presents with polyuria and dehydration.

A nurse should establish which outcomes on the care plan for a patient taking oral terbutaline? (Select all that apply.) A. Absence of tachycardia B. No reports of chest pain C. Less irritation of gum tissue D. Decrease in bronchospasm E. No jaundice or dark urine

A. Absence of tachycardia B. No reports of chest pain D. Decrease in bronchospasm Terbutaline is an oral beta2 agonist. It promotes bronchodilation through activation of beta2 receptors in the lung, which relieves bronchospasm. It also stimulates the beta receptors of the heart and can result in tachycardia and chest pain if dosing is excessive. Irritated mucosal tissue of the gums, jaundice, and dark urine are not adverse effects of the medication.

When teaching a patient regarding desmopressin (DDAVP), the nurse will inform the patient to monitor for which potential side effects? (Select all that apply.) A. Headache B. Weight gain C. Nasal irritation D. Hyperglycemia E. Hypotension

A. Headache Desmopressin works to decrease urine output; thus the patient would retain fluid and gain weight. Headache may also occur as a sequela of fluid retention. Because it is administered intranasally, it can be irritating; thus nostrils should be rotated. Desmopressin does not affect serum glucose levels.

Which are advantages of a dry-powder inhaler (DPI) over a metered-dose inhaler (MDI)? (Select all that apply.) A. More of the drug is delivered to the lungs and less to the oropharynx. B. Use of a spacer is not necessary. C. Less propellant is needed to deliver the medication. D. Less hand-lung coordination is required. E. DPIs pose no environmental risk.

A. More of the drug is delivered to the lungs and less to the oropharynx. B. Use of a spacer is not necessary. D. Less hand-lung coordination is required. E. DPIs pose no environmental risk. DPIs deliver more drug to the lungs (20% of the total released, compared to 10% for MDIs). Spacers are not necessary with DPIs; they are recommended for use with MDIs to increase the delivery of drug to the lungs rather than the oropharyngeal mucosa. DPIs do not require the hand-lung coordination needed with MDIs. DPIs present no environmental hazard, because no propellant is required for delivery.

A patient is experiencing severe diarrhea, flushing and life threatening hypotension associated with carcinoid crisis. The nurse will prepare to administer which drug? A. octreotide (Sandostatin) B. vasopressin (Pitressin) C. somatropin (Humatrope) D. cosyntropin (Cortrosyn)

A. Octreotide (Sandostatin)

Using a stepwise approach to managing asthma, a nurse teaches a patient who is at step 1 to use albuterol MDI [Proventil] at which of these times? A. Whenever needed (PRN) as a quick-relief agent B. Twice daily combined with an inhaled glucocorticoid C. Only with a long-acting beta2 agonist (LABA) D. If nighttime awakenings occur more than 2 days a week

A. Whenever needed (PRN) as a quick-relief agent For patients at step 1 in the stepwise approach to managing asthma, albuterol is a short-acting beta2 agonist (SABA) used only PRN to relieve ongoing asthma attacks and prevent exercise-induced bronchospasm. For patients at step 1, no long-term control medications are taken. A patient is at a higher step than 1 in the stepwise approach if the patient requires a daily inhaled glucocorticoid or LABA or awakens at night more often than 2 days a week. For patients at steps 2 to 6, albuterol is considered a quick-relief medication taken PRN.

The nurse is providing care to a patient following a non-accidental traumatic brain injury. The patient has developed diabetes insipidus due to the injury. What medication is most often used in the management of diabetes insipidus? A. desmopressin (DDAVP) B. corticotrophin (Acthar) C. octreotide (Sandostatin) D. somatropin (Humatrope)

A. desmopressin (DDAVP) Vasopressin (Pitressin) and desmopressin (DDAVP) are used to prevent or control polydipsia (excessive thirst), polyuria, and dehydration in patients with diabetes insipidus caused by a deficiency of endogenous antidiuretic hormone.

A patient with antidiuretic hormone deficiency is receiving desmopressin (DDAVP). The nurse will teach this patient to: a.avoid grapefruit juice. b.increase sodium intake. c.monitor blood pressure. d.reduce fluid intake.

ANS : D

1. A patient is to receive vasopressin (Pitressin) 5 units subcut BID. The medication is available in a vial that contains 20 units/mL. Identify how many milliliters will the nurse draw up for this dose. (record answer using two decimal places) _______

ANS: DIF: COGNITIVE LEVEL: Applying (Application) REF: N/A TOP: NURSING PROCESS: Implementation

1. The nurse is giving morning medications. The Medication Administration Record has an order for levothyroxine, 75 mcg PO. The drug-dispensing cabinet contains levothyroxine tablets in milligram strengths instead of micrograms. Calculate the milligram equivalent dose of 75 mcg. _______

ANS: DIF: COGNITIVE LEVEL: Applying (Application) REF: N/A TOP: NURSING PROCESS: Implementation

1. A patient has a metered-dose inhaler that contains 200 actuations ('puffs'), and it does not have a dose counter. He is to take two puffs two times a day. If he does not take any extra doses, identify how many days will this inhaler last at the prescribed dose. _______

ANS: 50 days Note the number of doses in the canister, and then calculate the number of days that the canister will last. For this question, assuming that two puffs are taken two times a day, and the inhaler has a capacity of 200 inhalations. Two puffs two times a day equal four inhalations per day. Four divided into 200 yields 50; that is, the inhaler will last approximately 50 days. DIF: COGNITIVE LEVEL: Applying (Application) REF: p. 594 TOP: NURSING PROCESS: Implementation

1. A patient will be receiving monthly injections of cyanocobalamin (Nascobal). The dose is 100 mcg/month IM. The medication is available in a strength of 1000 mcg/mL. Identify how many milliliters will the nurse draw up into the syringe. (record answer using one decimal place) _______

ANS: 0.1 mL DIF: COGNITIVE LEVEL: Applying (Application) REF: N/A TOP: NURSING PROCESS: Implementation

19. The nurse is teaching a group of patients about management of diabetes. Which statement about basal dosing is correct? a. "Basal dosing delivers a constant dose of insulin." b. "With basal dosing, you can eat what you want and then give yourself a dose of insulin." c. "Glargine insulin is given as a bolus with meals." d. "Basal-bolus dosing is the traditional method of managing blood glucose levels."

ANS: A Basal-bolus therapy is the attempt to mimic a healthy pancreas by delivering basal insulin constantly as a basal, and then as needed as a bolus. Glargine insulin is used as a basal dose, not as a bolus with meals. Basal-bolus therapy is a newer therapy; historically, sliding-scale coverage was implemented. DIF: COGNITIVE LEVEL: Applying (Application) REF: p. 508 TOP: NURSING PROCESS: Implementation

5. A nursing student is discussing with a nurse the plan of care for a patient about to undergo a third round of chemotherapy with cisplatin. Which statement by the nursing student about the treatment of CINV is correct? a. "Aprepitant [Emend] will be necessary to treat CINV caused by cisplatin." b. "Antiemetics are most effective if given just as the chemotherapy is finished." c. "Lorazepam probably would not be helpful for this patient." d. "This patient will need intravenous antiemetics for best effects.

ANS: A CINV caused by cisplatin is maximal 48 to 72 hours after dosing and can persist for 6 to 7 days, so an antiemetic such as aprepitant, which treats delayed emesis, is an important part of antiemetic therapy. Antiemetics should be started 30 minutes before initiation of chemotherapy. Lorazepam should be given to this patient, because the patient has had some experience with chemotherapy and is likely to have anticipatory emesis. There is no benefit to IV over oral dosing unless the patient has ongoing emesis.

1. A nurse is taking a history on a clinic patient who reports being constipated. Upon further questioning, the nurse learns that the patient's last stool was 4 days ago, that it was of normal, soft consistency, and that the patient defecated without straining. The patient's abdomen is not distended, and bowel sounds are present. The patient reports usually having a stool every 1 to 2 days. What will the nurse do? a. Ask about recent food and fluid intake. b. Discuss the use of polyethylene glycol [MiraLax]. c. Recommend a bulk laxative. d. Suggest using a bisacodyl [Dulcolax] suppository.

ANS: A Constipation cannot necessarily be defined by the frequency of bowel movements, because this varies from one individual to another. Constipation is defined in terms of a variety of symptoms, including hard stools, infrequent stools, excessive straining, prolonged effort, and unsuccessful or incomplete defecation. A common cause of constipation is diet, especially fluid and fiber intake; therefore, when changes in stool patterns occur, patients should be questioned about food and fluid intake. Because this patient has only more infrequent stools and is not truly constipated, laxatives are not indicated.

8. A patient will be starting vitamin D supplements. The nurse reviews his medical record for contraindications, including which condition? a. Renal disease b. Cardiac disease c. Hypophosphatemia d. There are no contraindications to vitamin D supplements.

ANS: A Contraindications to vitamin D products include known allergy to the product, hypercalcemia, renal dysfunction, and hyperphosphatemia. DIF: COGNITIVE LEVEL: Applying (Application) REF: p. 845 TOP: NURSING PROCESS: Assessment

6. A patient's medication order indicates that he is to receive a dose of cosyntropin (Cortrosyn). The nurse is aware that this drug is used to diagnose which condition? a. Adrenocortical insufficiency b. Diabetes insipidus c. Myasthenia gravis d. Pituitary dwarfism

ANS: A Cosyntropin is used for the diagnosis of adrenocortical insufficiency. The other options are incorrect. DIF: COGNITIVE LEVEL: Understanding (Comprehension) REF: p. 485 TOP: NURSING PROCESS: Planning

10. The nurse is discussing the use of adsorbents such as bismuth subsalicylate (Pepto-Bismol) with a patient who has diarrhea. The nurse will warn the patient about which possible adverse effects? a. Dark stools and blue gums b. Urinary hesitancy c. Drowsiness and dizziness d. Blurred vision and headache

ANS: A Dark stools and blue gums are two of the possible adverse effects of bismuth subsalicylate (see Table 51-2). The other adverse effects listed may occur with the use of other antidiarrheal drugs. DIF: COGNITIVE LEVEL: Understanding (Comprehension) REF: p. 815 TOP: NURSING PROCESS: Assessment

A patient is hospitalized with head trauma after a motor vehicle accident. The nurse caring for the patient notes a marked increase in the output of pale, dilute urine. The nurse suspects which condition? a.Diabetes insipidus b.Diabetes mellitus c.Syndrome of inappropriate antidiuretic hormone secretion (SIADH) d.Water intoxication

ANS: A Deficiency of antidiuretic hormone (ADH) produces hypothalamic diabetes insipidus, in which large volumes of dilute urine are produced. Head trauma can cause the hypothalamus to stop producing ADH. Diabetes mellitus is an endocrine disorder of the pancreas that causes the production of large volumes of nondilute urine. SIADH is a condition in which too much ADH is produced, causing oliguria. Water intoxication occurs with SIADH.

A nurse teaches a nursing student about the differences between esmopressin (DDAVP) and vasopressin [Pitressin]. Which statement by the student indicates a need for further teaching? a."Desmopressin has a shorter duration of action than vasopressin." b."Desmopressin is easier to administer than vasopressin." c."Vasopressin can be used in cardiac resuscitation." d."Vasopressin can cause serious adverse cardiovascular effects."

ANS: A Desmopressin has a long duration of action, which is the reason it is preferred for the treatment of diabetes insipidus. It can be administered intranasally, so it is easier to administer than vasopressin. Vasopressin has hemodynamic effects that can be beneficial during cardiac resuscitation but that also can cause serious adverse cardiovascular effects.

A patient with acromegaly asks the nurse about treatments for this ondition. What will the nurse tell the patient? a.Drugs are generally used after surgical and radiation therapies have been tried. b.Drug therapy is easy to administer. c.Drug therapy is inexpensive. d.Drug therapy is generally short term.

ANS: A Drugs for acromegaly are generally reserved for patients who do not respond to other therapies or for whom these therapies are not viable options. Drug therapy requires daily subQ injections and is expensive. Drug therapy is indefinite, not short term.

5. A 75-year-old woman with type 2 diabetes has recently been placed on glipizide (Glucotrol), 10 mg daily. She asks the nurse when the best time would be to take this medication. What is the nurse's best response? a. "Take this medication in the morning, 30 minutes before breakfast." b. "Take this medication in the evening with a snack." c. "This medication needs to be taken after the midday meal." d. "It does not matter what time of day you take this medication."

ANS: A Glipizide is taken in the morning, 30 minutes before breakfast. When taken at this time, it has a longer duration of action, causing a constant amount of insulin to be released. This may be beneficial in controlling blood glucose levels throughout the day. DIF: COGNITIVE LEVEL: Applying (Application) REF: p. 511 TOP: NURSING PROCESS: Implementation

4. The nurse assesses a newly diagnosed patient for short-term complications of diabetes. What does this assessment include? a. Evaluation for hyperglycemia, hypoglycemia, and ketoacidosis b. Cranial nerve testing for peripheral neuropathy c. Pedal pulse palpation for arterial insufficiency d. Auscultation of the carotids for bruits associated with atherosclerosis

ANS: A High blood sugar, low blood sugar, and ketoacidosis are short-term complications of diabetes. Microvascular and macrovascular complications, such as peripheral neuropathy, are long-term complications of diabetes. Arterial insufficiency and atherosclerosis also are long-term complications of diabetes.

7. An adolescent patient recently attended a health fair and had a serum glucose test. The patient telephones the nurse and says, "My level was 125 mg/dL. Does that mean I have diabetes?" What is the nurse's most accurate response? a. "Unless you were fasting for longer than 8 hours, this does not necessarily mean you have diabetes." b. "At this level, you probably have diabetes. You will need an oral glucose tolerance test this week." c. "This level is conclusive evidence that you have diabetes." d. "This level is conclusive evidence that you do not have diabetes."

ANS: A If a person has not fasted for 8 hours, a blood sugar level of 125 mg/dL would be considered normal, because it is less than 200 mg/dL for a random sampling. Also, a person must have positive outcomes on two separate days to be diagnosed with diabetes. This patient does not need to have an oral glucose tolerance test, because the 125 mg/dL reading is so far below 200 mg/dL, which would require further work-up. No conclusive evidence indicates that this patient has diabetes, because the random sample value is so low, and the patient has not had two separate tests on different days. However, this also is not conclusive evidence that the patient does not have diabetes.

7. A patient, newly diagnosed with hypothyroidism, has received a prescription for thyroid replacement therapy. The nurse will instruct the patient to take this medication at which time of day? a. In the morning b. With the noon meal c. With the evening meal d. At bedtime

ANS: A If possible, it is best to administer thyroid drugs taken once daily in the morning so as to decrease the likelihood of insomnia that may result from evening dosing. DIF: COGNITIVE LEVEL: Applying (Application) REF: p. 496 TOP: NURSING PROCESS: Implementation

4. An 8-year-old child has been diagnosed with true pituitary dwarfism and is being treated with somatropin. In follow-up visits, the nurse will monitor for which expected outcome? a. Increased growth b. Decreased urinary output c. Increased muscle strength d. Increased height when the child reaches puberty

ANS: A In patients for whom somatropin is indicated, increased growth is expected. The other options are incorrect. DIF: COGNITIVE LEVEL: Understanding (Comprehension) REF: p. 488 TOP: NURSING PROCESS: Evaluation

1. The nurse is reviewing conditions caused by nutrient deficiencies. Conditions such as infantile rickets, tetany, and osteomalacia are caused by a deficiency in which vitamin or mineral? a. Vitamin D b. Vitamin C c. Zinc d. Cyanocobalamin (vitamin B12)

ANS: A Infantile rickets, tetany, and osteomalacia are all a result of long-term vitamin D deficiency. The other options are incorrect. DIF: COGNITIVE LEVEL: Remembering (Knowledge) REF: p. 845 TOP: NURSING PROCESS: Assessment

15. A patient is receiving lactulose (Enulose) three times a day. The nurse knows that the patient is not constipated and is receiving this drug for which reason? a. High ammonia levels due to liver failure b. Prevention of constipation c. Chronic renal failure d. Chronic diarrhea

ANS: A Lactulose (Enulose) produces a laxative effect but also works to reduce blood ammonia levels by converting ammonia to ammonium. Ammonium is a water-soluble cation that is trapped in the intestines and cannot be reabsorbed into the systemic circulation. This effect has proved helpful in reducing elevated serum ammonia levels in patients with severe liver disease. The other options are incorrect. DIF: COGNITIVE LEVEL: Applying (Application) REF: p. 822 TOP: NURSING PROCESS: Planning

8. Which condition would cause the nurse to withhold a PRN order for magnesium hydroxide? a. Chronic renal failure b. Cirrhosis c. Hemorrhoids d. Prostatitis

ANS: A Magnesium can accumulate to toxic levels in patients with renal dysfunction. The nurse should withhold the medication. Magnesium hydroxide is not contraindicated for patients with hemorrhoids, prostatitis, or cirrhosis.

8. When administering mineral oil, the nurse recognizes that it can interfere with the absorption of which substance? a. Fat-soluble vitamins b. Water-soluble vitamins c. Minerals d. Electrolytes

ANS: A Mineral oil can decrease the absorption of fat-soluble vitamins (A, D, E, and K). The other options are incorrect. DIF: COGNITIVE LEVEL: Applying (Application) REF: p. 820 TOP: NURSING PROCESS: Planning

10. A patient with type 1 diabetes reports mixing NPH and regular insulin to allow for one injection. What should the nurse tell the patient? a. This is an acceptable practice. b. These two forms of insulin are not compatible and cannot be mixed. c. Mixing these two forms of insulin may increase the overall potency of the products. d. NPH insulin should only be mixed with insulin glargine.

ANS: A NPH insulin is the only insulin suitable for mixing with short-acting insulins, such as insulin aspart [NovoLog]. These insulins are compatible and are mixed frequently for management of diabetics. The overall potency of each insulin is not increased by mixing them. Insulin glargine cannot be mixed with any other insulin for administration.

6. A patient who is in her first trimester of pregnancy asks the nurse to recommend nonpharmaceutical therapies for morning sickness. What will the nurse suggest? a. Avoiding fatty and spicy foods b. Consuming extra clear fluids c. Eating three meals daily d. Taking foods later in the day

ANS: A Nausea and vomiting of pregnancy (NVP) can be treated with nondrug measures, including avoiding fatty and spicy foods. Consuming extra fluids does not help with nausea and vomiting (N/V) but may be needed to prevent dehydration. Patients should be advised to eat small portions of food throughout the day rather than three complete meals. "Morning sickness" may actually occur all day, so delaying intake is not recommended.

5. After receiving a nebulizer treatment with a beta agonist, the patient complains of feeling slightly nervous and wonders if her asthma is getting worse. What is the nurse's best response? a. "This is an expected adverse effect. Let me take your pulse." b. "The next scheduled nebulizer treatment will be skipped." c. "I will notify the physician about this adverse effect." d. "We will hold the treatment for 24 hours."

ANS: A Nervousness, tremors, and cardiac stimulation are possible and expected adverse effects of beta agonists. The other options are incorrect responses. DIF: COGNITIVE LEVEL: Applying (Application) REF: p. 583 TOP: NURSING PROCESS: Assessment

6. An adult who has been self-medicating, using nutritional therapy for an elevated cholesterol level, complains of repeated episodes of flushing. The nurse suspects that the patient has been taking: a. niacin. b. thiamine. c. riboflavin. d. pyridoxine.

ANS: A Niacin is used to reduce cholesterol levels. When taken in large doses, nicotinic acid can cause vasodilation, with resultant flushing, dizziness, and nausea. Flushing is not a side effect of thiamine, riboflavin, or pyridoxine because they do not cause vasodilation. PTS: 1 DIF: Cognitive Level: Application REF: pp. 992-993

5. When reviewing the medication profile of a patient with a new order for desmopressin (DDAVP), the nurse notes that a drug interaction will occur if which drug is taken with desmopressin? a. Aspirin b. Digoxin c. Lithium d. Penicillin

ANS: C Lithium may cause a decreased therapeutic effect of desmopressin. The other options are incorrect. DIF: COGNITIVE LEVEL: Understanding (Comprehension) REF: p. 486 TOP: NURSING PROCESS: Assessment

9. The nurse is caring for an older adult patient after a right hip open reduction internal fixation (ORIF). The patient is taking an opioid every 6 hours as needed for pain. The nurse discusses obtaining an order from the prescriber for which medication? a. Docusate sodium [Colace] b. GoLYTELY c. Lactulose d. Polyethylene glycol [MiraLax]

ANS: A Oxycodone can be constipating. The patient needs something prophylactically, such as docusate sodium, that can be taken daily to prevent constipation. In addition, the patient's mobility is limited, which can further increase the risk of constipation. GoLYTELY is not indicated for constipation; it is used for cleansing the bowel before diagnostic procedures. Lactulose is not indicated; it typically is used for reducing ammonia levels in hepatic encephalopathy. Polyethylene glycol is indicated for occasional constipation; no information suggests that the patient is constipated.

9. Which action is most appropriate regarding the nurse's administration of a rapid-acting insulin to a hospitalized patient? a. Give it within 15 minutes of mealtime. b. Give it after the meal has been completed. c. Administer it once daily at the time of the midday meal. d. Administer it with a snack before bedtime.

ANS: A Rapid-acting insulins, such as insulin lispro and insulin aspart, are able to mimic closely the body's natural rapid insulin output after eating a meal; for this reason, both insulins are usually administered within 15 minutes of the patient's mealtime. The other options are incorrect. DIF: COGNITIVE LEVEL: Applying (Application) REF: p. 505 TOP: NURSING PROCESS: Planning

3. A patient will be taking bismuth subsalicylate (Pepto-Bismol) to control diarrhea. When reviewing the patient's other ordered medications, the nurse recognizes that which medication or medication class will interact significantly with the Pepto-Bismol? a. Hypoglycemic drugs b. Antibiotics c. Acetaminophen (Tylenol) d. Antidepressants

ANS: A Taking hypoglycemic drugs with an adsorbent such as bismuth subsalicylate may result in decreased absorption of the hypoglycemic drugs. The other options are incorrect. DIF: COGNITIVE LEVEL: Understanding (Comprehension) REF: p. 816 TOP: NURSING PROCESS: Implementation

1. The nurse working on a high-acuity medical-surgical unit is prioritizing care for four patients who were just admitted. Which patient should the nurse assess first? a. The NPO patient with a blood glucose level of 80 mg/dL who just received 20 units of 70/30 Novolin insulin b. The patient with a pulse of 58 beats per minute who is about to receive digoxin [Lanoxin] c. The patient with a blood pressure of 136/92 mm Hg who complains of having a headache d. The patient with an allergy to penicillin who is receiving an infusion of vancomycin [Vancocin]

ANS: A The NPO patient with hypoglycemia who just received 70/30 Novolin insulin takes priority, because this patient needs to consume a good source of glucose immediately or perhaps the NPO status will be discontinued for this shift. The digoxin may be withheld for the patient with a pulse of 58 beats per minute, but this is not a priority action. The patient with a headache needs to be followed up, but because the blood pressure is 136/92 mm Hg, the headache is probably not caused by hypertension. The patient with an allergy to penicillin will not have a reaction to the vancomycin.

8. A nurse explains to a nursing student why opioid antidiarrheal medications are classified as drugs with little or no abuse potential. Which statement by the student indicates a need for further teaching? a. "Formulations for the treatment of diarrhea have very short half-lives." b. "Opioid antidiarrheal drugs contain other drugs with unpleasant side effects at higher doses." c. "Some opioid antidiarrheal drugs do not cross the blood-brain barrier." d. "Some opioid antidiarrheal medications are not water soluble and cannot be given parenterally."

ANS: A The half-life of the opioid antidiarrheal drugs is the same as that of the opioid analgesics. The formulations of opioid antidiarrheal medications that are classified with low abuse potential are often combined with atropine, which has unpleasant side effects at higher doses. Some opioid antidiarrheal drugs are formulated so that they do not cross the blood-brain barrier. Others are not water soluble and therefore cannot be dissolved and injected.

1. The nurse is administering insulin lispro (Humalog) and will keep in mind that this insulin will start to have an effect within which time frame? a. 15 minutes b. 1 to 2 hours c. 80 minutes d. 3 to 5 hours

ANS: A The onset of action for insulin lispro is 15 minutes. The peak plasma concentration is 1 to 2 hours; the elimination half-life is 80 minutes; and the duration of action is 3 to 5 hours. DIF: COGNITIVE LEVEL: Remembering (Knowledge) REF: p. 516 TOP: NURSING PROCESS: Implementation

A male patient reports decreased libido and the nurse notes galactorrhea during a physical assessment. The nurse will report these findings to the provider and will anticipate an order for which medication? a.Cabergoline b.Conivaptan c.Dopamine d.Prolactin

ANS: A The patient is showing signs of prolactin hypersecretion. Cabergoline is a dopamine agonist and is used to inhibit prolactin release. Conivaptan is used to treat hypernatremia. Dopamine is not given. Prolactin would make the condition worse.

2. A patient will be starting therapy with a corticosteroid. The nurse reviews the patient's orders and notes that an interaction may occur if the corticosteroid is taken with which of these drug classes? a. Nonsteroidal anti-inflammatory drugs b. Antibiotics c. Opioid analgesics d. Antidepressants

ANS: A The use of corticosteroids with aspirin, other NSAIDs, and other ulcerogenic drugs produces additive gastrointestinal effects and an increased chance for the development of gastric ulcers. The other options are incorrect. DIF: COGNITIVE LEVEL: Understanding (Comprehension) REF: p. 526 TOP: NURSING PROCESS: Assessment

13. Which statement is correct about the contrast between acarbose and miglitol? a. Miglitol has not been associated with hepatic dysfunction. b. With miglitol, sucrose can be used to treat hypoglycemia. c. Miglitol is less effective in African Americans. d. Miglitol has no gastrointestinal side effects.

ANS: A Unlike acarbose, miglitol has not been associated with hepatic dysfunction. Sucrose should not be used to treat hypoglycemia with miglitol. Miglitol is more effective in African American patients. Miglitol has gastrointestinal side effects.

4. The nurse is assessing a patient who is malnourished and has a history of poor nutrition. The patient reports difficulty seeing at night. This patient is likely to be deficient in which fat-soluble vitamin? a. A (retinol) b. D c. E (alpha-tocopherol) d. K

ANS: A Vitamin A is needed for dark adaptation; night blindness often is the first indication of deficiency. Night blindness is not a sign of deficiency of vitamins D, E, or K. PTS: 1 DIF: Cognitive Level: Application REF: pp. 988-989

Which types of drugs are used to treat inflammatory bowel disease (IBD)? (Select all that apply.) a. Aminosalicylates b. Glucocorticoids c. Immunomodulators d. Opioid antidiarrheals e. Sulfonamide antibiotics

ANS: A, B, C Five types of drugs are used to treat IBD: aminosalicylates, glucocorticoids, immunosuppressants, immunomodulators, and antibiotics such as metronidazole and ciprofloxacin. Opioid antidiarrheal drugs and sulfonamide antibiotics are not used for this purpose.

1. A patient is taking a sulfonylurea medication for new-onset type 2 diabetes mellitus. When reviewing potential adverse effects during patient teaching, the nurse will include information about which of these effects? (Select all that apply.) a. Hypoglycemia b. Nausea c. Diarrhea d. Weight gain e. Peripheral edema

ANS: A, B, D The most common adverse effect of the sulfonylureas is hypoglycemia, the degree to which depends on the dose, eating habits, and presence of hepatic or renal disease. Another predictable adverse effect is weight gain because of the stimulation of insulin secretion. Other adverse effects include skin rash, nausea, epigastric fullness, and heartburn. DIF: COGNITIVE LEVEL: Applying (Application) REF: p. 509 TOP: NURSING PROCESS: Implementation

3. During an intravenous infusion of calcium, the nurse carefully monitors the patient for symptoms of hypercalcemia. Which are symptoms of hypercalcemia? (Select all that apply.) a. Anorexia b. Nausea and vomiting c. Diarrhea d. Constipation e. Cardiac irregularities f. Drowsiness

ANS: A, B, D, E Symptoms of hypercalcemia include anorexia, nausea, vomiting, and constipation. Long-term excessive calcium intake can result in severe hypercalcemia, which can cause cardiac irregularities, delirium, and coma. The other options are incorrect. DIF: COGNITIVE LEVEL: Applying (Application) REF: p. 853 TOP: NURSING PROCESS: Implementation

1. Levothyroxine (Synthroid) has been prescribed for a patient with hypothyroidism. The nurse reviews the patient's current medications for potential interactions. Which of these drugs or drug classes interact with levothyroxine? (Select all that apply.) a. Phenytoin (Dilantin) b. Estrogens c. Beta blockers d. Warfarin (Coumadin) e. Penicillins f. Iron supplements

ANS: A, B, D, F Drug interactions with thyroid preparations include phenytoin, cholestyramine, antacids, calcium salts, iron products, estrogens, and warfarin (see Table 31-3). The other options are not correct. DIF: COGNITIVE LEVEL: Applying (Application) REF: p. 493 TOP: NURSING PROCESS: Planning

2. The nurse expects that a patient is experiencing undersecretion of adrenocortical hormones when which conditions are found upon assessment? (Select all that apply.) a. Dehydration b. Weight loss c. Steroid psychosis d. Increased potassium levels e. Increased blood glucose levels f. Decreased serum sodium levels

ANS: A, B, D, F The undersecretion (hyposecretion) of adrenocortical hormones causes a condition known as Addison's disease, which is associated with decreased blood sodium and glucose levels, increased potassium levels, dehydration, and weight loss. Steroid psychosis is an effect of glucocorticoid excess. DIF: COGNITIVE LEVEL: Applying (Application) REF: p. 524 TOP: NURSING PROCESS: Assessment

Which are indications for discontinuing growth hormone (GH) therapy in children with documented growth hormone deficiency? (Select all that apply.) a.Epiphyseal closure has begun. b.A satisfactory adult height has been achieved. c.Serum GH levels have been normal for 12 consecutive months. d.Normal serum growth hormone levels occur. e.The child no longer responds to the hormone.

ANS: A, B, E Treatment with GH for a child with a documented GH deficiency may continue until a satisfactory adult height has been achieved, if a child does not show a response to the hormone, or until epiphyseal closure begins. Treatment does not continue for life, and GH should not be administered during or after closure of the epiphyses. Treatment does not depend on growth hormone levels; these are measured to ensure therapeutic dosing.

1. Which of the following vitamins can be stored by the body? (Select all that apply.) a. Vitamin E (alpha-tocopherol) b. Vitamin C (ascorbic acid) c. Cyanocobalamin (vitamin B12) d. Folic acid e. Vitamin D

ANS: A, C, E The fat-soluble vitamins (A, D, E, and K) are stored in massive amounts in the body. Of the water-soluble vitamins, B12 can be stored; all others must be replenished with frequent ingestion. PTS: 1 DIF: Cognitive Level: Comprehension REF: pp. 987-988

1. The nurse is reviewing vitamin therapy in preparation for a nutrition class. Which statements are accurate regarding vitamin C (ascorbic acid)? (Select all that apply.) a. Vitamin C is important in the maintenance of bone, teeth, and capillaries. b. Vitamin C is essential for night vision. c. Vitamin C is important for tissue repair. d. Vitamin C is found in animal sources such as dairy products and meat. e. Vitamin C is found in tomatoes, strawberries, and broccoli. f. Vitamin C is also known as the "sunshine vitamin." g. Vitamin C deficiency is known as scurvy.

ANS: A, C, E, G These statements are true of vitamin C. Vitamin A is essential for night vision, and vitamin D is known as the sunshine vitamin. With the exception of liver, meat and dairy products are not sources of vitamin C. DIF: COGNITIVE LEVEL: Applying (Application) REF: p. 852 TOP: NURSING PROCESS: Implementation

9. The nurse is reviewing the mechanism of action of antidiarrheal drugs. Which type of antidiarrheal medication works by decreasing the intestinal muscle tone and peristalsis of the intestines? a. Adsorbents such as Pepto-Bismol b. Anticholinergics such as belladonna alkaloids c. Probiotics such as Lactinex d. Lubricants such as mineral oil

ANS: B Anticholinergic drugs work to slow peristalsis by reducing the rhythmic contractions and the smooth muscle tone of the gastrointestinal tract. The other options are incorrect. DIF: COGNITIVE LEVEL: Understanding (Comprehension) REF: p. 815 TOP: NURSING PROCESS: Planning

2. What are the potential beneficial effects of taking vitamin E (alpha-tocopherol) supplements? (Select all that apply.) a. Delayed progression of macular degeneration b. Improved blood clotting c. Lowered risk of developing Alzheimer's disease d. Reduced risk of cardiovascular disease e. Reduced risk of colds in elderly patients

ANS: A, E Studies to determine the potential benefits of vitamin E supplements show a decreased risk of colds in elderly patients when vitamin E is combined with vitamin C, beta-carotene, zinc, and copper, and a potential delay in the progression of macular degeneration. Vitamin E may actually increase the risk of bleeding. Studies have not shown a decrease in the risk of Alzheimer's disease or cardiovascular disease. PTS: 1 DIF: Cognitive Level: Analysis REF: pp. 990-991

3. A nurse is educating the staff nurses about ketoacidosis. To evaluate the group's understanding, the nurse asks, "Which sign or symptom would not be consistent with ketoacidosis?" The group gives which correct answer? a. Blood glucose level of 600 mg/dL b. Blood glucose level of 60 mg/dL c. Acidosis d. Ketones in the urine

ANS: B A patient with diabetic ketoacidosis (DKA) has a high glucose level (at least 500 mg/dL or higher); therefore, a glucose level of 60 mg/dL would not be consistent with DKA. A blood glucose level of 600 mg/dL, acidosis, and ketones in the urine are consistent with DKA.

1. The nurse is administering adrenal drugs to a patient. Which action by the nurse is appropriate for this patient? a. Administering oral drugs on an empty stomach to maximize absorption b. Rinsing the oral cavity after using corticosteroid inhalers c. Administering the corticosteroids before bedtime to minimize adrenal suppression d. Discontinuing the medication immediately if weight gain of 5 pounds or more in 1 week occurs

ANS: B After the patient has used the corticosteroid inhalers, cleaning the oral cavity helps to prevent possible oral fungal infections from developing. Adrenal drugs need be taken with meals to minimize gastrointestinal upset and in the mornings to minimize adrenal suppression, and they need to be discontinued by weaning, not abruptly. DIF: COGNITIVE LEVEL: Applying (Application) REF: p. 530 TOP: NURSING PROCESS: Implementation

12. A nurse is providing teaching to a nursing student about to care for a woman with irritable bowel syndrome with diarrhea (IBS-D) who is receiving alosetron [Lotronex]. Which statement by the student indicates a need for further teaching? a. "I should evaluate the patient's abdomen for distension and bowel sounds." b. "Patients with diverticulitis and IBS-C may take this drug." c. "This drug can cause ischemic colitis in some patients." d. "This drug is given only to women with severe IBS-D."

ANS: B Alosetron is approved for use in women only with diarrhea-predominant IBS; it is contraindicated in patients with diverticulitis. Constipation can be a severe adverse effect, so patients should be assessed for signs of constipation, such as abdominal distension and diminished bowel sounds. Alosetron can cause ischemic colitis. Alosetron is approved for use in women with IBS-D regardless of the severity of the disease.

6. A patient asks the nurse about the difference between diphenoxylate with atropine (Lomotil) and the over-the-counter drug loperamide (Imodium). Which response by the nurse is correct? a. "Lomotil acts faster than Imodium." b. "Imodium does not cause physical dependence." c. "Lomotil is available in suppository form." d. "Imodium is a natural antidiarrheal drug."

ANS: B Although the drug exhibits many characteristics of the opiate class, physical dependence on loperamide has not been reported. All antidiarrheal drugs are orally administered. The other options are incorrect. DIF: COGNITIVE LEVEL: Applying (Application) REF: p. 817 TOP: NURSING PROCESS: Planning

6. A patient has prescriptions for two inhalers. One inhaler is a bronchodilator, and the other is a corticosteroid. Which instruction regarding these inhalers will the nurse give to the patient? a. "Take the corticosteroid inhaler first." b. "Take the bronchodilator inhaler first." c. "Take these two drugs at least 2 hours apart." d. "It does not matter which inhaler you use first."

ANS: B An inhaled bronchodilator is used before the inhaled corticosteroid to provide bronchodilation before administration of the anti-inflammatory drug. DIF: COGNITIVE LEVEL: Applying (Application) REF: p. 592 TOP: NURSING PROCESS: Implementation

8. When reviewing the laboratory values of a patient who is taking antithyroid drugs, the nurse will monitor for which adverse effect? a. Decreased glucose levels b. Decreased white blood cell count c. Increased red blood cell count d. Increased platelet count

ANS: B Antithyroid drugs may cause bone marrow suppression, resulting in agranulocytosis, leukopenia, thrombocytopenia, and other problems. The other options are incorrect. DIF: COGNITIVE LEVEL: Applying (Application) REF: p. 495 TOP: NURSING PROCESS: Evaluation

9. A patient is taking bismuth subsalicylate [Pepto-Bismol] to prevent diarrhea. The nurse performing an assessment notes that the patient's tongue is black. What will the nurse do? a. Assess further for signs of gastrointestinal (GI) bleeding. b. Reassure the patient that this is an expected side effect of this drug. c. Request an order for liver function tests to evaluate for hepatotoxicity. d. Withhold the drug, because this is a sign of bismuth overdose.

ANS: B Bismuth subsalicylate can cause blackening of the tongue and stools, an expected side effect. This finding does not indicate GI bleeding, hepatotoxicity, or drug overdose.

7. A patient wants to prevent problems with constipation and asks the nurse for advice about which type of laxative is safe to use for this purpose. Which class of laxative is considered safe to use on a long-term basis? a. Emollient laxatives b. Bulk-forming laxatives c. Hyperosmotic laxatives d. Stimulant laxatives

ANS: B Bulk-forming laxatives are the only laxatives recommended for long-term use. Stimulant laxatives are the most likely of all the laxative classes to cause dependence. The other options are incorrect. DIF: COGNITIVE LEVEL: Applying (Application) REF: p. 819 TOP: NURSING PROCESS: Planning

14. A nurse receives an order to administer castor oil to a patient. Which action by the nurse is correct? a. Administer the medication at bedtime. b. Chill the medication and mix it with fruit juice. c. Provide teaching about home use of this medication. d. Teach the patient that the effects will occur slowly.

ANS: B Castor oil has an unpleasant taste that can be improved by chilling it and mixing it with fruit juice. The medication acts quickly and should not be given at bedtime. It is only used when prompt evacuation of intestinal contents is needed, as for radiological procedures, so the patient will not be instructed in home use of the medication and should be taught that the effects will be immediate.

6. A glucocorticoid is prescribed for a patient. The nurse checks the patient's medical history knowing that glucocorticoid therapy is contraindicated in which disorder? a. Cerebral edema b. Peptic ulcer disease c. Tuberculous meningitis d. Chronic obstructive pulmonary disease

ANS: B Contraindications to the administration of glucocorticoids include drug allergy and may include cataracts, glaucoma, peptic ulcer disease, mental health problems, and diabetes mellitus. The other options are indications for glucocorticoids. DIF: COGNITIVE LEVEL: Understanding (Comprehension) REF: p. 526 TOP: NURSING PROCESS: Assessment

13. A patient with a history of chronic obstructive pulmonary disease (COPD) and type 2 diabetes has been treated for pneumonia for the past week. The patient has been receiving intravenous corticosteroids as well as antibiotics as part of his therapy. At this time, the pneumonia has resolved, but when monitoring the blood glucose levels, the nurse notices that the level is still elevated. What is the best explanation for this elevation? a. The antibiotics may cause an increase in glucose levels. b. The corticosteroids may cause an increase in glucose levels. c. His type 2 diabetes has converted to type 1. d. The hypoxia caused by the COPD causes an increased need for insulin.

ANS: B Corticosteroids can antagonize the hypoglycemic effects of insulin, resulting in elevated blood glucose levels. The other options are incorrect. DIF: COGNITIVE LEVEL: Analyzing (Analysis) REF: p. 515 TOP: NURSING PROCESS: Evaluation

14. A nurse is discussing the use of immunosuppressants for the treatment of inflammatory bowel disease (IBD) with a group of nursing students. Which statement by a student indicates understanding of the teaching? a. "Azathioprine [Imuran] helps induce rapid remission of IBD." b. "Cyclosporine [Sandimmune] can be used to induce remission of IBD." c. "Cyclosporine [Sandimmune] does not have serious adverse effects." d. "Methotrexate is used long term to maintain remission of IBD."

ANS: B Cyclosporine can be given intravenously to induce rapid remission of IBD. Azathioprine has delayed onset of effects up to 6 months and is not used to induce rapid remission. Cyclosporine is a toxic compound that can cause renal impairment, neurotoxicity, and immune suppression. Methotrexate is used to promote short-term remission.

1. The nurse notes in a patient's medication history that the patient has been taking desmopressin (DDAVP). Based on this finding, the nurse interprets that the patient has which disorder? a. Diabetes mellitus b. Diabetes insipidus c. Adrenocortical insufficiency d. Carcinoid tumor

ANS: B Desmopressin is used to prevent or control polydipsia (excessive thirst), polyuria, and dehydration in patients with diabetes insipidus. The symptoms are caused by a deficiency of endogenous antidiuretic hormone. The other options are incorrect. DIF: COGNITIVE LEVEL: Understanding (Comprehension) REF: p. 485 TOP: NURSING PROCESS: Assessment

3. A patient is concerned about the body changes that have resulted from long-term prednisone therapy for the treatment of asthma. Which effect of this drug therapy would be present to support the nursing diagnosis of disturbed body image? a. Weight loss b. Weight gain c. Pale skin color d. Hair loss

ANS: B Facial erythema, weight gain, hirsutism, and "moon face" (characteristic of Cushing's syndrome) are possible body changes that may occur with long-term prednisone therapy. DIF: COGNITIVE LEVEL: Analyzing (Analysis) REF: p. 527 TOP: NURSING PROCESS: Nursing Diagnosis

15. A patient has been diagnosed with metabolic syndrome and is started on the biguanide metformin (Glucophage). The nurse knows that the purpose of the metformin, in this situation, is which of these? a. To increase the pancreatic secretion of insulin b. To decrease insulin resistance c. To increase blood glucose levels d. To decrease the pancreatic secretion of insulin

ANS: B Metformin decreases glucose production by the liver; decreases intestinal absorption of glucose; and improves insulin receptor sensitivity in the liver, skeletal muscle, and adipose tissue, resulting in decreased insulin resistance. The other options are incorrect. DIF: COGNITIVE LEVEL: Understanding (Comprehension) REF: p. 508 TOP: NURSING PROCESS: Planning

17. The nurse is preparing to administer methylnaltrexone (Relistor), a peripherally acting opioid antagonist. This drug is appropriate for which patient? a. A patient with diarrhea b. A terminally ill patient who has opioid-induced constipation c. A patient who is scheduled for a colonoscopy d. A patient who will be having colon surgery in the morning

ANS: B Methylnaltrexone is approved only for terminally ill (hospice) patients who have opioid-induced constipation. The other options are incorrect. DIF: COGNITIVE LEVEL: Applying (Application) REF: p. 819 TOP: NURSING PROCESS: Planning

7. A patient who has been on long-term corticosteroid therapy has had surgery to correct an abdominal hernia. The nurse keeps in mind that which potential effect of this medication may have the most impact on the patient's recovery? a. Hypotension b. Delayed wound healing c. Muscle weakness d. Osteoporosis

ANS: B Muscle weakness and osteoporosis may also result from long-term therapy, but delayed wound healing would have the most impact on the patient's recovery from abdominal surgery at this time. Hypertension, not hypotension, may result from long-term corticosteroid therapy. DIF: COGNITIVE LEVEL: Applying (Application) REF: p. 526 TOP: NURSING PROCESS: Planning

14. A nursing student asks a nurse about the recommendations for vitamin supplements to prevent cancer. Which statement by the student indicates a need for further teaching? a. "Foods rich in antioxidant vitamins may confer protection by other means." b. "Observational studies provide clear proof that some antioxidants reduce the risk of cancer." c. "Randomized controlled studies have not demonstrated cancer prevention benefits." d. "Some vitamins have an increased cancer risk associated with too low and too high amounts."

ANS: B No clear evidence indicates that antioxidant vitamins lead to a reduced risk of cancer. Some observational studies show a link between the consumption of foods high in antioxidants and lower cancer rates, but this link may be related to other components of these diets, such as the high fiber also found in such foods. Some vitamins are linked to higher cancer rates when consumed in both too-low and too-high amounts. PTS: 1 DIF: Cognitive Level: Analysis REF: p. 990

10. A patient will undergo a colonoscopy, and the provider has ordered sodium phosphate as a bowel cleanser before the procedure. The nurse reviews the patient's chart and notes that the patient's creatinine clearance and blood urea nitrogen are both elevated. What will the nurse do? a. Reduce the amount of fluid given with the laxative to prevent fluid retention. b. Request an order to give polyethylene glycol and electrolytes (PEG-ELS) instead. c. Suggest that the patient reduce the dietary sodium intake. d. Suggest using a suppository laxative instead.

ANS: B PEG-ELS solutions provide an isosmotic solution and do not cause dehydration or electrolyte imbalance. They are safe to use in patients with renal impairment or cardiovascular disease. This patient's laboratory values suggest renal impairment. Sodium phosphate products can cause kidney damage; giving them with less fluid only increases this possibility. Reducing the dietary intake of sodium is not recommended. Suppositories are not effective bowel cleansing agents.

6. A 19-year-old student was diagnosed with hypothyroidism and has started thyroid replacement therapy with levothyroxine (Synthroid). After 1 week, she called the clinic to report that she does not feel better. Which response from the nurse is correct? a. "It will probably require surgery for a cure to happen." b. "The full therapeutic effects may not occur for 3 to 4 weeks." c. "Is it possible that you did not take your medication as instructed?" d. "Let's review your diet; it may be causing absorption problems."

ANS: B Patients need to understand that it may take up to 3 to 4 weeks to see the full therapeutic effects of thyroid drugs. The other options are incorrect. DIF: COGNITIVE LEVEL: Applying (Application) REF: p. 497 TOP: NURSING PROCESS: Evaluation

4. During a teaching session for a patient on antithyroid drugs, the nurse will discuss which dietary instructions? a. Using iodized salt when cooking b. Avoiding foods containing iodine c. Restricting fluid intake to 2500 mL/day d. Increasing intake of sodium- and potassium-containing foods

ANS: B Patients on antithyroid therapy need to avoid iodine-containing foods. These foods may interfere with the effectiveness of the antithyroid drug. The other options are incorrect. DIF: COGNITIVE LEVEL: Applying (Application) REF: p. 496 TOP: NURSING PROCESS: Implementation

4. A patient is taking fludrocortisone (Florinef) for Addison's disease, and his wife is concerned about all of the problems that may occur with this therapy. When teaching them about therapy with this drug, the nurse will include which information? a. It may cause severe postural hypotension. b. It needs to be taken with food or milk to minimize gastrointestinal upset. c. The medication needs to be stopped immediately if nausea or vomiting occurs. d. Weight gain of 5 pounds or more in 1 week is an expected adverse effect.

ANS: B Patients receiving fludrocortisone need to take it with food or milk to minimize gastrointestinal upset; weight gain of 5 pounds or more in 1 week needs to be reported to the physician; abrupt withdrawal is not recommended because it may precipitate an adrenal crisis. Adverse effects are related to the fluid retention and may include heart failure and hypertension. DIF: COGNITIVE LEVEL: Applying (Application) REF: p. 531 TOP: NURSING PROCESS: Implementation

5. A patient who is taking propylthiouracil (PTU) for hyperthyroidism wants to know how this medicine works. Which explanation by the nurse is accurate? a. It blocks the action of thyroid hormone. b. It slows down the formation of thyroid hormone. c. It destroys overactive cells in the thyroid gland. d. It inactivates already existing thyroid hormone in the bloodstream.

ANS: B Propylthiouracil impedes the formation of thyroid hormone but has no effect on already existing thyroid hormone. The other options are incorrect. DIF: COGNITIVE LEVEL: Understanding (Comprehension) REF: p. 494 TOP: NURSING PROCESS: Implementation

12. The nurse is preparing to administer insulin intravenously. Which statement about the administration of intravenous insulin is true? a. Insulin is never given intravenously. b. Only regular insulin can be administered intravenously. c. Insulin aspart or insulin lispro can be administered intravenously, but there must be a 50% dose reduction. d. Any form of insulin can be administered intravenously at the same dose as that is ordered for subcutaneous administration.

ANS: B Regular insulin is the usual insulin product to be dosed via intravenous bolus, intravenous infusion, or even intramuscularly. These routes, especially the intravenous infusion route, are often used in cases of diabetic ketoacidosis, or coma associated with uncontrolled type 1 diabetes. DIF: COGNITIVE LEVEL: Applying (Application) REF: p. 516 TOP: NURSING PROCESS: Implementation

17. The nurse is reviewing a patient's medication list and notes that sitagliptin (Januvia) is ordered. The nurse will question an additional order for which drug or drug class? a. Glitazone b. Insulin c. Metformin (Glucophage) d. Sulfonylurea

ANS: B Sitagliptin is indicated for management of type 2 diabetes either as monotherapy or in combination with metformin, a sulfonylurea, or a glitazone, but not with insulin. DIF: COGNITIVE LEVEL: Applying (Application) REF: p. 512 TOP: NURSING PROCESS: Implementation

13. The nurse is providing education to a patient with ulcerative colitis who is being treated with sulfasalazine [Azulfidine]. What statement by the patient best demonstrates understanding of the action of sulfasalazine? a. "It treats the infection that triggers the condition." b. "It reduces the inflammation." c. "It enhances the immune response." d. "It increases the reabsorption of fluid."

ANS: B Sulfasalazine reduces the inflammation seen with ulcerative colitis; this statement indicates understanding. Although similar to sulfonamides, sulfasalazine is not used to treat infections; further teaching is needed. Sulfasalazine does not enhance the immune response or increase the reabsorption of fluid; further teaching is needed.

5. When monitoring a patient who is taking a systemically administered glucocorticoid, the nurse will monitor for signs of which condition? a. Dehydration b. Hypokalemia c. Hyponatremia d. Hypoglycemia

ANS: B Systemic glucocorticoid drugs may cause potassium depletion, hyperglycemia, and hypernatremia. The other options are incorrect. DIF: COGNITIVE LEVEL: Understanding (Comprehension) REF: p. 526 TOP: NURSING PROCESS: Evaluation

2. A patient is taking a xanthine derivative as part of treatment for chronic obstructive pulmonary disease. The nurse will monitor for which adverse effects associated with the use of xanthine derivatives? a. Diarrhea b. Palpitations c. Bradycardia d. Drowsiness

ANS: B The common adverse effects of the xanthine derivatives include nausea, vomiting, and anorexia. In addition, gastroesophageal reflux has been observed to occur during sleep in patients taking these drugs. Cardiac adverse effects include sinus tachycardia, extrasystole, palpitations, and ventricular dysrhythmias. Transient increased urination and hyperglycemia are other possible adverse effects. DIF: COGNITIVE LEVEL: Remembering (Knowledge) REF: p. 585 TOP: NURSING PROCESS: Evaluation

11. A patient reports taking an oral bisacodyl laxative [Dulcolax] for several years. The provider has suggested discontinuing the laxative, but the patient is unsure how to do this. The nurse will tell the patient to: a. stop taking the oral laxative and use a suppository until normal motility resumes. b. stop taking the laxative immediately and expect no stool for several days. c. switch to a bulk-forming laxative, such as methylcellulose [Metamucil]. d. withdraw from the laxative slowly to avoid a rebound constipation effect.

ANS: B The first step in breaking the laxative habit is abrupt cessation of laxative use. Bowel movements will be absent for several days after laxative withdrawal. Using a suppository, a bulk-forming laxative, or tapering the laxative only prolongs the habit and prevents normal function from returning.

10. After starting treatment for type 2 diabetes mellitus 6 months earlier, a patient is in the office for a follow-up examination. The nurse will monitor which laboratory test to evaluate the patient's adherence to the antidiabetic therapy over the past few months? a. Hemoglobin levels b. Hemoglobin A1C level c. Fingerstick fasting blood glucose level d. Serum insulin levels

ANS: B The hemoglobin A1C level reflects the patient's adherence to the therapy regimen for several months previously, thus evaluating how well the patient has been doing with diet and drug therapy. The other options are incorrect. DIF: COGNITIVE LEVEL: Applying (Application) REF: p. 517 TOP: NURSING PROCESS: Evaluation

A nurse is preparing to administer a dose of growth hormone and reconstitutes the medication. After adding the diluent, the nurse notices that the preparation is cloudy. What will the nurse do? a.Administer the drug as ordered. b.Discard the drug and prepare another dose. c.Notify the prescriber. d.Shake the drug to dissipate the particles.

ANS: B The medication should not be injected if the preparation is cloudy or contains particulate matter. The drug should be discarded and another dose drawn up and administered. There is no need to notify the prescriber. The preparation should never be shaken

2. A pregnant patient who is taking ondansetron [Zofran] for morning sickness tells the nurse she is experiencing headache and dizziness. What will the nurse tell her? a. It is not safe to take this drug during pregnancy. b. These are common side effects of ondansetron. c. She should stop taking the ondansetron immediately. d. She should report these adverse effects to her provider.

ANS: B The most common side effects of ondansetron are headache, diarrhea, and dizziness. Ondansetron is used off-label to treat morning sickness. These side effects do not indicate a need to stop taking the drug or to report the side effects to the provider.

21. The insulin order reads, "Give 10 units of NPH insulin and 5 units of regular insulin, subcut, every morning before breakfast." Choose the proper syringe for this injection. a. b. c. d.

ANS: B The proper syringe for insulin injection is the insulin syringe, which is marked in units. The other syringes listed are not correct for use with insulin because they are not marked in units. DIF: COGNITIVE LEVEL: Analyzing (Analysis) REF: p. 506 TOP: NURSING PROCESS: Planning

3. A patient is in an urgent care center with an acute asthma attack. The nurse expects that which medication will be used for initial treatment? a. An anticholinergic such as ipratropium (Atrovent) b. A short-acting beta2 agonist such as albuterol (Proventil) c. A long-acting beta2 agonist such as salmeterol (Serevent) d. A corticosteroid such as fluticasone (Flovent)

ANS: B The short-acting beta2 agonists are commonly used during the acute phase of an asthmatic attack to reduce airway constriction quickly and to restore airflow to normal levels. The other drugs listed are not appropriate for acute asthma attacks. Anticholinergic drugs and long-acting beta2 agonists are used to prevent attacks; corticosteroids are used to reduce airway inflammation. DIF: COGNITIVE LEVEL: Applying (Application) REF: p. 582 TOP: NURSING PROCESS: Planning

3. A patient is on vitamin D supplemental therapy. The nurse will monitor for which signs of toxicity during this therapy? a. Tinnitus b. Anorexia c. Diarrhea d. Hypotension

ANS: B The toxic effects of vitamin D are those associated with hypertension, such as weakness, fatigue, headache, anorexia, dry mouth, metallic taste, nausea, vomiting, abdominal cramps, ataxia, and bone pain. If not recognized and treated, these symptoms can progress to impairment of renal function and osteoporosis. The other options listed are not signs of vitamin D toxicity. DIF: COGNITIVE LEVEL: Applying (Application) REF: p. 845 TOP: NURSING PROCESS: Assessment

7. A patient who experiences motion sickness is about to go on a cruise. The prescriber orders transdermal scopolamine [Transderm Scop]. The patient asks the nurse why an oral agent is not ordered. The nurse will explain that the transdermal preparation: a. can be applied as needed at the first sign of nausea. b. has less intense anticholinergic effects than the oral form. c. is less sedating than the oral preparation. d. provides direct effects, because it is placed close to the vestibular apparatus of the ear.

ANS: B The transdermal system of scopolamine is preferred, because it may have less intense anticholinergic effects than oral or subcutaneous dosing. Antinausea medications for motion sickness are more effective if given prophylactically than after symptoms begin. Sedation side effects are similar with all forms. Placement near the ear does not cause the medication to absorb directly into the vestibular apparatus.

16. When administering morning medications for a newly admitted patient, the nurse notes that the patient has an allergy to sulfa drugs. There is an order for the sulfonylurea glipizide (Glucotrol). Which action by the nurse is correct? a. Give the drug as ordered 30 minutes before breakfast. b. Hold the drug, and check the order with the prescriber. c. Give a reduced dose of the drug with breakfast. d. Give the drug, and monitor for adverse effects.

ANS: B There is a potential for cross-allergy in patients who are allergic to sulfonamide antibiotics. Although such an allergy is listed as a contraindication by the manufacturer, most clinicians do prescribe sulfonylureas for such patients. The order needs to be clarified. DIF: COGNITIVE LEVEL: Applying (Application) REF: p. 509 TOP: NURSING PROCESS: Implementation

6. A patient with a history of alcohol abuse has been admitted for severe weakness and malnutrition. The nurse will prepare to administer which vitamin preparation to prevent Wernicke's encephalopathy? a. Vitamin B3 (niacin) b. Vitamin B1 (thiamine) c. Vitamin B6 (pyridoxine) d. Folic acid

ANS: B Thiamine is necessary for the treatment of a variety of thiamine deficiencies, including Wernicke's encephalopathy. The other options are incorrect. DIF: COGNITIVE LEVEL: Understanding (Comprehension) REF: p. 848 TOP: NURSING PROCESS: Planning

10. A patient who is malnourished has scaling, cracked skin on the arms and face. The patient is irritable, anxious, and has difficulty sleeping. The patient complains of soreness of the tongue and mouth. When teaching this patient about vitamin therapy for this disorder, the nurse will tell the patient to report which side effects? a. Abdominal cramps and diarrhea b. Flushing, dizziness, and nausea c. Migraine headaches d. Numbness of feet and hands

ANS: B This patient shows signs of niacin deficiency. Side effects of niacin deficiency include flushing, dizziness, and nausea secondary to vasodilation. Abdominal cramps, diarrhea, migraines, and numbness of extremities are not side effects of niacin deficiency. PTS: 1 DIF: Cognitive Level: Analysis REF: pp. 992-993

3. When a patient is receiving vasopressin (Pitressin), the nurse will monitor for which therapeutic response? a. Improved appetite b. Increased serum albumin levels c. Increased serum potassium levels d. Decreased urinary output

ANS: D Decreased severe thirst and decreased urinary output are the therapeutic responses expected with vasopressin. The other options are incorrect. DIF: COGNITIVE LEVEL: Understanding (Comprehension) REF: p. 487 TOP: NURSING PROCESS: Evaluation

2. The nurse is preparing a plan of care for a patient undergoing therapy with vitamin A. Which nursing diagnosis is appropriate for this patient? a. Impaired tissue integrity related to vitamin deficiency b. Risk for injury related to night blindness caused by vitamin deficiency c. Impaired physical mobility (muscle weakness) related to vitamin deficiency d. Acute confusion related to vitamin deficiency

ANS: B Vitamin A deficiency causes night blindness, so risk for injury is an appropriate nursing diagnosis. The other nursing diagnoses are not appropriate for patients receiving vitamin A. DIF: COGNITIVE LEVEL: Applying (Application) REF: p. 857 TOP: NURSING PROCESS: Nursing Diagnosis

14. The nurse knows to administer acarbose (Precose), an alpha-glucosidase inhibitor, at which time? a. 30 minutes before breakfast b. With the first bite of each main meal c. 30 minutes after breakfast d. Once daily at bedtime

ANS: B When an alpha-glucosidase inhibitor is taken with the first bite of a meal, excessive postprandial blood glucose elevation (a glucose spike) can be reduced or prevented. DIF: COGNITIVE LEVEL: Applying (Application) REF: p. 510 TOP: NURSING PROCESS: Implementation

10. A newly admitted patient has orders for a zinc supplement. The nurse reviews the patient's medical history and concludes that the zinc is ordered for which reason? a. To treat pellagra b. To aid in wound healing c. To treat osteomalacia d. As an antidote for anticoagulant overdose

ANS: B Zinc plays a crucial role in the enzymatic metabolic reactions involving both proteins and carbohydrates. This makes it especially important for normal tissue growth and repair. It therefore also has a major role in wound healing. Vitamin B3 (niacin) is used to treat pellagra; vitamin D is used to treat osteomalacia; and vitamin K is used as an antidote for anticoagulant overdose. DIF: COGNITIVE LEVEL: Applying (Application) REF: p. 856 TOP: NURSING PROCESS: Planning

1. The nurse is reviewing medications for the treatment of asthma. Which drugs are used for acute asthma attacks? (Select all that apply.) a. Salmeterol (Serevent) inhaler b. Albuterol (Proventil) nebulizer solution c. Epinephrine d. Montelukast (Singulair) e. Fluticasone (Flovent) Rotadisk inhaler

ANS: B, C Albuterol (a short-acting beta2 agonist) and epinephrine (a beta1 and beta2 agonist) are used for acute bronchospasms. Salmeterol is a long-acting beta2 agonist that is indicated for maintenance treatment, not acute episodes. Fluticasone is an inhaled corticosteroid; montelukast is a leukotriene receptor antagonist (LTRA). These types of medications are used for asthma prophylaxis. DIF: COGNITIVE LEVEL: Applying (Application) REF: p. 591 TOP: NURSING PROCESS: Implementation

1. The nurse is reviewing therapy with glucocorticoid drugs. Which conditions are indications for glucocorticoid drugs? (Select all that apply.) a. Glaucoma b. Cerebral edema c. Chronic obstructive pulmonary disease and asthma d. Organ transplantation e. Varicella f. Septicemia

ANS: B, C, D Cerebral edema, chronic obstructive pulmonary disease, asthma, and organ transplantation are indications for glucocorticoid therapy. Glaucoma, varicella, and septicemia are all contraindications to glucocorticoid therapy. DIF: COGNITIVE LEVEL: Applying (Application) REF: p. 525 TOP: NURSING PROCESS: Planning

Which laxatives may be used safely in infants? (Select all that apply.) a. Castor oil b. Docusate sodium (Colace) c. Glycerin suppositories d. Lactulose e. Milk of magnesia

ANS: B, C, D Docusate, lactulose, and glycerin suppositories have been used to treat constipation safely in infants. Castor oil and milk of magnesia are not.

What are the effects of prolactin secretion in males? (Select all that apply.) a.Breast development b.Decreased libido c.Delayed puberty d.Galactorrhea e.Infertility

ANS: B, C, D Prolactin can cause decreased libido, delayed puberty, and galactorrhea in males, but it does not cause breast development or infertility.

5. The nurse will prepare to give which preparation to a newborn upon arrival in the nursery after delivery? a. Vitamin B1 (thiamine) b. Vitamin D (calciferol) c. Folic acid d. Vitamin K (AquaMEPHYTON)

ANS: D Deficiency in vitamin K can be seen in newborns because of malabsorption attributed to inadequate amounts of bile. AquaMEPHYTON is given as a single intramuscular dose for infants upon arrival in the nursery. DIF: COGNITIVE LEVEL: Understanding (Comprehension) REF: p. 847 TOP: NURSING PROCESS: Implementation

Which patients would be candidates for the use of dronabinol [Marinol] to treat nausea and vomiting? (Select all that apply.) a. A patient with a history of a psychiatric disorder b. A patient with acquired immunodeficiency syndrome (AIDS)-induced anorexia c. A patient with chemotherapy-induced nausea and vomiting d. A patient with nausea who has used marijuana in the past e. A patient with postoperative nausea and vomiting

ANS: B, C, D Two cannabinoids have been approved for medical use in the United States for AIDS-induced anorexia and CINV. Because these drugs do not produce the same "high" that smoking marijuana does, their use is not contraindicated in patients who have used marijuana in the past. They do cause potentially unpleasant effects, such as temporal disintegration, dissociation, depersonalization, and dysphoria, so they are contraindicated in patients with psychiatric disorders. They are not approved for postoperative nausea and vomiting.

2. The patient asks the nurse about taking large doses of vitamin C to improve her immunity to colds. "It's just a vitamin, right? What can happen?" Which responses by the nurse are correct? (Select all that apply.) a. "Vitamin C is harmless because it is a water-soluble vitamin." b. "Large doses of vitamin C can cause nausea, vomiting, headache, and abdominal cramps." c. "Keep in mind that if you suddenly stop taking these large doses, you might experience symptoms similar to scurvy." d. "Studies have shown that vitamin C has little value in preventing the common cold." e. "Vitamin C acidifies the urine, which can lead to the formation of kidney stones." f. "Large doses of vitamin C may delay wound healing."

ANS: B, C, D, E Vitamin C is usually nontoxic unless excessive dosages are consumed. Large doses (megadoses) can produce nausea, vomiting, headache, and abdominal cramps, and they acidify the urine, which can result in the formation of kidney stones. Furthermore, individuals who discontinue taking excessive daily doses of ascorbic acid can experience scurvy-like symptoms. Studies have shown that megadoses of vitamin C have little or no value as prophylaxis against the common cold. Vitamin C is required for several important metabolic activities, including collagen synthesis and the maintenance of connective tissue and tissue repair. DIF: COGNITIVE LEVEL: Applying (Application) REF: p. 852 TOP: NURSING PROCESS: Implementation

1. The nurse is reviewing the uses of oral laxatives. Which conditions are general contraindications to or cautions about the use of oral laxatives? (Select all that apply.) a. Irritable bowel syndrome b. Undiagnosed abdominal pain c. Nausea and vomiting d. Fecal impaction e. Ingestion of toxic substances f. Acute surgical abdomen

ANS: B, C, D, F Cautious use of laxatives is recommended in the presence of these: acute surgical abdomen; appendicitis symptoms, such as abdominal pain, nausea, and vomiting; intestinal obstruction; and undiagnosed abdominal pain. Oral laxatives must not be used with fecal impaction; mineral oil enemas are indicated for fecal impaction. The other options are indications for laxative use. DIF: COGNITIVE LEVEL: Applying (Application) REF: p. 820 TOP: NURSING PROCESS: Assessment

2. The nurse is providing instructions to a patient who has a new prescription for a corticosteroid metered-dose inhaler. Which statement by the patient indicates that further instruction is needed? (Select all that apply.) a. "I will rinse my mouth with water after using the inhaler and then spit out the water." b. "I will gargle after using the inhaler and then swallow." c. "I will clean the plastic inhaler casing weekly by removing the canister and then washing the casing in warm soapy water. I will then let it dry before reassembling." d. "I will use this inhaler for asthma attacks." e. "I will continue to use this inhaler, even if I am feeling better." f. "I will use a peak flow meter to measure my response to therapy."

ANS: B, D The inhaled corticosteroid is a maintenance drug used to prevent asthma attacks; it is not indicated for acute asthma attacks. Rinsing the mouth with water is appropriate and necessary to prevent oral fungal infections; the water is not to be swallowed after rinsing. The patient needs to be given instructions about keeping the inhaler clean, including removing the canister from the plastic casing weekly and washing the casing in warm soapy water. Once the casing is dry, the canister and mouthpiece may be put back together and the cap applied. The glucocorticoid may predispose the patient to oral fungal overgrowth, thus the need for implicit instructions about cleaning inhaling devices. Use of a peak flow meter assists in monitoring the patient's response to therapy. The medication needs to be taken as ordered every day, regardless of whether the patient is feeling better. DIF: COGNITIVE LEVEL: Applying (Application) REF: p. 594 TOP: NURSING PROCESS: Implementation

A patient newly diagnosed with diabetes expresses concern about losing her vision. Which interventions should be included in the plan of care to reduce this risk? (Select all that apply.) a. Initiation of reliable contraception to prevent pregnancy b. Ways to reduce hyperglycemic episodes c. Use of a prokinetic drug (eg, metoclopramide) d. Smoking cessation e. Emphasis on the importance of taking antihypertensive drugs consistently

ANS: B, D, E Tighter glycemic control and fewer hyperglycemic episodes have been correlated with fewer complications. Smoking cessation and strict compliance with any drugs prescribed for hypertension also help reduce the risk of complications. The use of contraception or a prokinetic drug is not related to a reduced risk of vision loss in patients with diabetes.

A nurse counsels a patient with diabetes who is starting therapy with an alpha-glucosidase inhibitor. The patient should be educated about the potential for which adverse reactions? (Select all that apply.) a. Hypoglycemia b. Flatulence c. Elevated iron levels in the blood d. Fluid retention e. Diarrhea

ANS: B, E Acarbose and miglitol frequently cause flatulence, cramps, abdominal distention, borborygmus (rumbling bowel sounds), and diarrhea. Acarbose also can cause liver damage. Neither acarbose nor miglitol causes hypoglycemia, elevated iron levels, or fluid retention.

6. A patient with type 1 diabetes recently became pregnant. The nurse plans a blood glucose testing schedule for her. What is the recommended monitoring schedule? a. Before each meal and before bed b. In the morning for a fasting level and at 4:00 PM for the peak level c. Six or seven times a day d. Three times a day, along with urine glucose testing

ANS: C A pregnant patient with type 1 diabetes must have frequent blood sugar monitoring (eg, six or seven times a day) to manage both the patient and the fetus so that no teratogenic effects occur. Monitoring the blood sugar level before meals and at bedtime is not significant enough to provide the necessary glycemic control. Morning and 4:00 PM monitoring is not enough to provide glycemic control. Urine glucose testing is not sensitive enough to aid glycemic control, and monitoring three times a day is not enough.

12. A patient who is known to be a heavy drinker is brought to the emergency department with ataxia and confusion. The patient cannot remember the events of the previous day. The examination reveals nystagmus, and the patient reports having double vision. The nurse will expect to administer which vitamin to this patient? a. Ascorbic acid (vitamin C) IV b. Intramuscular pyridoxine (vitamin B6) c. Intravenous thiamine (vitamin B1) d. Nicotinic acid (niacin) PO

ANS: C Alcoholics who are malnourished have a form of thiamine deficiency called Wernicke-Korsakoff syndrome, which is characterized by nystagmus, diplopia, ataxia, confusion, and short-term memory loss. Parenteral thiamine is indicated for treatment. Ascorbic acid is given to treat vitamin C deficiency, or scurvy. Pyridoxine is given for vitamin B6 deficiency, also common in alcoholics, but this patient does not have the symptoms of seborrheic dermatitis and peripheral neuropathy. Nicotinic acid is used for niacin deficiency, which is characterized by severe dry, rough skin. PTS: 1 DIF: Cognitive Level: Application REF: p. 993

13. A laxative has been ordered for a patient. The nurse checks the patient's medical history and would be concerned if which condition is present? a. High ammonia levels due to liver failure b. Diverticulosis c. Abdominal pain of unknown origin d. Chronic constipation

ANS: C All categories of laxatives share the same general contraindications and precautions, including avoidance in cases of drug allergy and the need for cautious use in the presence of these: acute surgical abdomen; appendicitis symptoms such as abdominal pain, nausea, and vomiting; fecal impaction (mineral oil enemas excepted); intestinal obstruction; and undiagnosed abdominal pain. The other options are possible indications for laxatives. DIF: COGNITIVE LEVEL: Applying (Application) REF: p. 820 TOP: NURSING PROCESS: Implementation

18. A patient has been treated with alosetron (Lotronex) for severe irritable bowel syndrome (IBS) for 2 weeks. She calls the clinic and tells the nurse that she has been experiencing constipation for 3 days. The nurse will take which action? a. Advise the patient to increase intake of fluids and fiber. b. Advise the patient to hold the drug for 2 days. c. Instruct the patient to stop taking the drug and to come to the clinic right away to be evaluated. d. Instruct the patient to continue the alosetron and to take milk of magnesia for the constipation.

ANS: C Alosetron must be discontinued immediately if constipation or signs of ischemic colitis occur. The other options are incorrect. DIF: COGNITIVE LEVEL: Analyzing (Analysis) REF: p. 823 TOP: NURSING PROCESS: Implementation

8. The nurse is monitoring drug levels for a patient who is receiving theophylline. The most recent theophylline level was 22 mcg/mL, and the nurse evaluates this level to be: a. below the therapeutic level. b. at a therapeutic level. c. above the therapeutic level. d. at a toxic level.

ANS: C Although the optimal level may vary from patient to patient, most standard references have suggested that the therapeutic range for theophylline blood level is 10 to 20 mcg/mL. However, most clinicians now advise levels between 5 and 15 mcg/mL. DIF: COGNITIVE LEVEL: Applying (Application) REF: p. 585 TOP: NURSING PROCESS: Evaluation

2. When administering a bulk-forming laxative, the nurse instructs the patient to drink the medication mixed in a full 8-ounce glass of water. Which statement best explains the rationale for this instruction? a. The water acts to stimulate bowel movements. b. The water will help to reduce the bulk of the intestinal contents. c. These laxatives may cause esophageal obstruction if taken with insufficient water. d. The water acts as a lubricant to produce bowel movements.

ANS: C Bulk-forming drugs increase water absorption, which results in greater total volume (bulk) of the intestinal contents. Bulk-forming laxatives tend to produce normal, formed stools. Their action is limited to the gastrointestinal tract, so there are few, if any, systemic effects. However, they need to be taken with liberal amounts of water to prevent esophageal obstruction and fecal impaction. DIF: COGNITIVE LEVEL: Applying (Application) REF: p. 820 TOP: NURSING PROCESS: Implementation

5. A patient's provider has recommended a bulk-forming laxative for occasional constipation. Which statement by the patient indicates understanding of the teaching about this agent? a. "I can take this medication long term." b. "I should not take this drug if I have diverticulitis." c. "I should take each dose with a full glass of water." d. "This drug can cause severe diarrhea."

ANS: C Bulk-forming laxatives provide insoluble substances that swell in water to both soften and increase the size of the fecal mass. Patients should be taught to take the dose with a full glass of water. Laxatives in general are not recommended for long-term use. Bulk-forming laxatives are safe for patients with diverticulitis. They are often used to treat diarrhea, because they help form the fecal mass.

16. A patient is taking linaclotide (Linzess) to treat irritable bowel syndrome (IBS). The nurse will monitor this patient for which adverse effect? a. Chest pain b. Chronic constipation c. Abdominal pain d. Elevated blood glucose levels

ANS: C Common adverse effects of linaclotide (Linzess) are diarrhea, abdominal pain, and flatulence. Elevated blood glucose levels, chest pain, and chronic constipation are not adverse effects of linaclotide. DIF: COGNITIVE LEVEL: Applying (Application) REF: p. 823 TOP: NURSING PROCESS: Assessment

2. When teaching about hypoglycemia, the nurse will make sure that the patient is aware of the early signs of hypoglycemia, including: a. hypothermia and seizures. b. nausea and diarrhea. c. confusion and sweating. d. fruity, acetone odor to the breath.

ANS: C Early symptoms of hypoglycemia include the central nervous system manifestations of confusion, irritability, tremor, and sweating. Hypothermia and seizures are later symptoms of hypoglycemia. The other options are incorrect. DIF: COGNITIVE LEVEL: Understanding (Comprehension) REF: p. 514 TOP: NURSING PROCESS: Implementation

14. An elderly patient who has type 2 diabetes has a history of severe hypoglycemia. The patient's spouse asks the nurse what the optimum A1c level is for the patient. Which is correct? a. Between 6.5 and 7.0 b. Below 7.0 c. Below 8.0 d. Between 7.0 and 8.5

ANS: C For patients with a history of severe hypoglycemia and those with a limited life expectancy or advanced micro- and macrovascular complications, the target A1c level should be below 8.0. For most other patients with diabetes, the target is 7.0 and below.

11. Insulin glargine is prescribed for a hospitalized patient who has diabetes. When will the nurse expect to administer this drug? a. Approximately 15 to 30 minutes before each meal b. In the morning and at 4:00 PM c. Once daily at bedtime d. After meals and at bedtime

ANS: C Glargine insulin is indicated for once daily subcutaneous administration to treat adults and children with type 1 diabetes and adults with type 2 diabetes. According to the package labeling, the once-daily injection should be given at bedtime. Glargine insulin should not be given more than once a day, although some patients require bid dosing to achieve a full 24 hours of basal coverage.

15. A patient who has type 2 diabetes will begin taking glipizide [Glucotrol]. Which statement by the patient is concerning to the nurse? a. "I will begin by taking this once daily with breakfast." b. "It is safe to drink grapefruit juice while taking this drug." c. "I may continue to have a glass of wine with dinner." d. "I will need to check my blood sugar once daily or more."

ANS: C Glipizide is a sulfonylurea antidiabetic agent and can cause a disulfiram-like reaction when combined with alcohol. Patients should be taught to avoid alcohol while taking this medication. The initial dosing is once daily with breakfast. There is no drug interaction with grapefruit juice. Patients will need to monitor their blood glucose.

A nurse is providing teaching for a nondiabetic adult who develops growth hormone deficiency and who will begin treatment with somatropin [Humatrope]. Which statement by the patient indicates understanding of the teaching? a."Intramuscular dosing is more effective than subcutaneous dosing." b."I will have increased muscle mass and strength as well as increased height." c."I will need to monitor my blood pressure frequently while taking this drug." d."I will need to take insulin while using this, because it causes hyperglycemia."

ANS: C Growth hormone in adults causes an increase in systolic blood pressure, so patients should be taught to monitor blood pressure while taking the drug. Subcutaneous dosing is as effective as IM dosing and is preferred, because it is less painful. Although muscle mass will increase, strength and height will not. Growth hormone is diabetogenic but causes significant problems in patients with preexisting diabetes

7. A 6-year-old child has frequent constipation. The nurse provides teaching after the parent asks the nurse why the provider recommended using laxatives only when needed. Which statement by the parent indicates a need for further teaching? a. "Children who take laxatives regularly can become dehydrated." b. "Chronic laxative use can cause electrolyte imbalances." c. "Frequent use of laxatives can cause diverticulitis." d. "The normal reflex to defecate can be inhibited with overuse of laxatives."

ANS: C Laxatives do not cause diverticulitis, although some laxatives can aggravate this condition. Chronic use of laxatives can cause dehydration and electrolyte imbalances and can suppress the normal defecation reflex.

7. The nurse is reviewing instructions for a patient with type 2 diabetes who also takes insulin injections as part of the therapy. The nurse asks the patient, "What should you do if your fasting blood glucose is 47 mg/dL?" Which response by the patient reflects a correct understanding of insulin therapy? a. "I will call my doctor right away." b. "I will give myself the regular insulin." c. "I will take an oral form of glucose." d. "I will rest until the symptoms pass."

ANS: C Hypoglycemia can be reversed if the patient eats glucose tablets or gel, corn syrup, or honey, or drinks fruit juice or a nondiet soft drink or other quick sources of glucose, which must always be kept at hand. She should not wait for instructions from her physician, nor delay taking the glucose by resting. The regular insulin would only lower her blood glucose levels more. DIF: COGNITIVE LEVEL: Applying (Application) REF: p. 518 TOP: NURSING PROCESS: Implementation

4. A patient is receiving intravenous promethazine [Phenergan] 25 mg for postoperative nausea and vomiting. What is an important nursing action when giving this drug? a. Giving the dose as an IV push over 3 to 5 minutes b. Infusing the dose with microbore tubing and an infusion pump c. Observing the IV insertion site frequently for patency d. Telling the patient to report dry mouth and sedation

ANS: C If IV administration must be done with this drug, it should be given through a large-bore, freely flowing line. The site should be monitored closely for local burning or pain or any sign of extravasation, which can cause abscess formation, tissue necrosis, and gangrene requiring amputation. Giving the medication as a rapid IV push or through microbore tubing does not adequately slow the infusion or dilute the drug. Dry mouth and sedation are expected side effects of this drug and are not dangerous.

11. A patient in the emergency department was showing signs of hypoglycemia and had a fingerstick glucose level of 34 mg/dL. The patient has just become unconscious. What is the nurse's next action? a. Have the patient eat glucose tablets. b. Have the patient consume fruit juice, a nondiet soft drink, or crackers. c. Administer intravenous glucose (50% dextrose). d. Call the lab to order a fasting blood glucose level.

ANS: C Intravenous glucose raises blood glucose levels when the patient is unconscious and unable to take oral forms of glucose. DIF: COGNITIVE LEVEL: Analyzing (Analysis) REF: p. 514 TOP: NURSING PROCESS: Implementation

12. A patient with a history of chronic alcohol abuse has been admitted to the unit with cirrhosis. Upon review of the patient's laboratory test results, the nurse notes that the patient's ammonia level is elevated at 218 mcg/dL. What medication should the nurse prepare to administer? a. 0.9% NS b. Docusate sodium [Colace] c. Lactulose d. Polyethylene glycol [MiraLax]

ANS: C Lactulose is the only laxative known to lower ammonia levels in patients with portal hypertension and hepatic encephalopathy secondary to liver disease. No information suggests that the patient needs fluid or electrolyte replacement. Docusate sodium and polyethylene glycol are not effective at lowering ammonia levels.

2. A patient is admitted with lower abdominal pain and nausea. The nurse performing the initial assessment notes that the patient's abdomen is distended and firm, and hypoactive bowel sounds are present. The patient has not had a stool for 3 days. The nurse will contact the provider, who will: a. order a bulk-forming laxative. b. order extra fluids and fiber. c. perform diagnostic tests. d. prescribe a cathartic laxative.

ANS: C Laxatives are contraindicated for patients with abdominal pain, nausea, cramps, or other symptoms of abdominal disease or an acute surgical abdomen. Laxatives should not be used in patients with obstruction or impaction. This patient shows signs of abdominal obstruction, and laxatives could cause a bowel perforation secondary to increased peristalsis. A bulk-forming laxative is contraindicated. Patients with acute abdomens should be kept NPO pending diagnosis. A cathartic laxative is contraindicated.

1. The nurse is giving oral mineral oil as an ordered laxative dose. The nurse will take measures to prevent which potential problem that may occur with mineral oil? a. Fecal impaction b. Electrolyte imbalances c. Lipid pneumonia d. Esophageal blockage

ANS: C Lipid pneumonia may occur if the oral mineral oil is accidentally aspirated into the respiratory tract. The other options are incorrect. DIF: COGNITIVE LEVEL: Applying (Application) REF: p. 820 TOP: NURSING PROCESS: Implementation

5. While recovering from surgery, a 74-year-old woman started taking a stimulant laxative, senna (Senokot), to relieve constipation caused by the pain medications. Two weeks later, at her follow-up appointment, she tells the nurse that she likes how "regular" her bowel movements are now that she is taking the laxative. Which teaching principle is appropriate for this patient? a. She needs to be sure to take this medication with plenty of fluids. b. It is important to have a daily bowel movement to promote bowel health. c. Long-term use of laxatives often results in decreased bowel tone and may lead to dependency. d. She needs to switch to glycerin suppositories to continue having daily bowel movements.

ANS: C Long-term use of laxatives may lead to dependency. Patients need to be taught that daily bowel movements are not necessary for bowel health. DIF: COGNITIVE LEVEL: Applying (Application) REF: p. 819 TOP: NURSING PROCESS: Implementation

7. Niacin is prescribed for a patient who has hyperlipidemia. The nurse checks the patient's medical history, knowing that this medication is contraindicated in which disorder? a. Renal disease b. Cardiac disease c. Liver disease d. Diabetes mellitus

ANS: C Niacin, unlike certain other B-complex vitamins, has additional contraindications besides drug allergy. They include liver disease, severe hypotension, arterial hemorrhage, and active peptic ulcer disease. The other options are incorrect. DIF: COGNITIVE LEVEL: Applying (Application) REF: p. 849 TOP: NURSING PROCESS: Assessment

9. A patient with hyperlipidemia has been told by the provider to take extra niacin. The nurse will tell the patient to: a. increase servings of poultry, fish, and cereals. b. take nicotinamide supplements. c. take nicotinic acid supplements. d. take tryptophan supplements.

ANS: C Nicotinic acid is given in high doses to treat hyperlipidemia. Increasing dietary niacin is not sufficient to treat hyperlipidemia. Nicotinamide does not affect plasma lipoproteins. Dietary tryptophan is converted by the body into nicotinic acid, but not in sufficient amounts to treat hyperlipidemia. PTS: 1 DIF: Cognitive Level: Application REF: pp. 992-993

15. A patient with Crohn's disease will begin receiving an initial infusion of infliximab [Remicade]. The nurse explains how this drug works to treat this disease. Which statement by the patient indicates a need for further teaching? a. "I may have an increased risk of infections, such as tuberculosis, when taking infliximab." b. "I should report chills, fever, itching, and shortness of breath while receiving the infusion." c. "This drug sometimes provides a complete cure of inflammatory bowel disease." d. "I will take the second dose in 2 weeks, the third dose in 6 weeks, and then a dose every 8 weeks thereafter."

ANS: C None of the drugs used to treat obstructive bowel disease (OBD) are curative. Patients taking immunomodulators, such as infliximab, have an increased risk of infection, especially opportunistic infections such as TB. Infusion reactions may occur and include chills, fever, itching, and shortness of breath. The induction regimen is 5 mg/kg infused at 0, 2, and 6 weeks, followed by a maintenance regimen every 8 weeks.

1. A nurse caring for a patient who is undergoing a third round of chemotherapy is preparing to administer ondansetron [Zofran] 30 minutes before initiation of the chemotherapy. The patient tells the nurse that the ondansetron did not work as well the last time as it had the first time. What will the nurse do? a. Administer the ondansetron at the same time as the chemotherapy. b. Contact the provider to suggest using high-dose intravenous dolasetron [Anzemet]. c. Request an order to administer dexamethasone with the ondansetron. d. Suggest to the provider that loperamide [Lomotil] be given with the ondansetron.

ANS: C Ondansetron is a serotonin receptor antagonist; drugs in this class are the most effective drugs available for suppressing nausea and vomiting associated with anticancer drugs. The drug is even more effective when combined with dexamethasone. For best effect, ondansetron should be given 30 minutes before beginning chemotherapy. Dolasetron is similar to ondansetron, but when given intravenously in high doses, it is associated with fatal dysrhythmias. Loperamide is used to treat diarrhea.

7. A patient will begin taking iron supplements to treat anemia. The nurse will recommend that the patient take the iron with which food to facilitate absorption? a. Cereal b. Dairy products c. Orange juice d. Red meats

ANS: C Orange juice is a good source of vitamin C, and vitamin C facilitates the absorption of iron. Cereals often are fortified with iron but do not facilitate its absorption. Calcium interferes with the absorption of iron. Red meats are a natural source of iron. PTS: 1 DIF: Cognitive Level: Analysis REF: p. 992

3. A patient with renal disease is scheduled for a colonoscopy. Before the procedure, the nurse will anticipate administering: a. glycerin suppository. b. magnesium hydroxide (MOM). c. polyethylene glycol and electrolytes. d. sodium phosphate.

ANS: C Polyethylene glycol (PEG) plus electrolytes (ELS) is one of two bowel cleansers used before colonoscopy to clear the bowel. PEG-ELS products are preferred, because unlike sodium phosphate, they are isotonic and do not increase the likelihood of dehydration and electrolyte imbalance. Glycerin suppositories and magnesium hydroxide are not used for bowel cleansing.

10. A patient has a diagnosis of primary hypothyroidism. Which statement accurately describes this problem? a. The hypothalamus is not secreting thyrotropin-releasing hormone (TRH); therefore, thyroid-stimulating hormone (TSH) is not released from the pituitary gland. b. The pituitary gland is dysfunctional and is not secreting TSH. c. The abnormality is in the thyroid gland itself. d. The abnormality is caused by an insufficient intake of iodine.

ANS: C Primary hypothyroidism stems from an abnormality in the thyroid gland itself and occurs when the thyroid gland is not able to perform one of its many functions. Secondary hypothyroidism begins at the level of the pituitary gland and results from reduced secretion of TSH. TSH is needed to trigger the release of the T3 and T4 stored in the thyroid gland. Tertiary hypothyroidism is caused by a reduced level of the TRH from the hypothalamus. This reduced level, in turn, reduces TSH and thyroid hormone levels. DIF: COGNITIVE LEVEL: Understanding (Comprehension) REF: p. 492 TOP: NURSING PROCESS: Assessment

11. Which B vitamin deficiency is associated with cheilosis, glossitis, vascularization of the cornea, and itchy dermatitis of the scrotum and vulva? a. Niacin (nicotinic acid) b. Pyridoxine (vitamin B6) c. Riboflavin (vitamin B2) d. Thiamine (vitamin B1)

ANS: C Riboflavin deficiency produces the symptoms described and can be treated with riboflavin supplements. These signs do not indicate deficiencies of niacin, pyridoxine, or thiamine. PTS: 1 DIF: Cognitive Level: Application REF: p. 993

14. A patient is severely constipated and needs immediate relief. The nurse knows that which class of laxative will provide the most rapid results? a. Bulk-forming laxative, such as psyllium (Metamucil) b. Stool softener, such as docusate salts (Colace) c. Magnesium hydroxide (MOM) d. Magnesium oxide tablets

ANS: C Saline laxatives such as magnesium hydroxide (MOM) produce a watery stool, usually within 3 to 6 hours of ingestion. Bulk-forming laxatives such as psyllium do not produce a bowel movement rapidly. Stool softeners such as docusate salts do not cause patients to defecate; they simply soften the stool to ease its passage. Magnesium oxide tablets are used as magnesium supplements, not as laxatives. DIF: COGNITIVE LEVEL: Understanding (Comprehension) REF: p. 822 TOP: NURSING PROCESS: Planning

2. A 16-year-old boy who is taking somatropin comes into the office because he had an asthma attack during a race at school. Because of this new development, the nurse expects which intervention to occur next? a. He will need to stop participating in school physical education classes. b. The somatropin must be discontinued immediately. c. The somatropin dosage may be adjusted. d. His growth will be documented and monitored for changes.

ANS: C Somatropin is to be used with caution in acute or chronic illnesses, such as migraine headaches, epilepsy, and asthma. It will not have to be immediately discontinued but will require close monitoring. The patient's growth will be measured and documented throughout therapy with somatropin. DIF: COGNITIVE LEVEL: Applying (Application) REF: p. 485 TOP: NURSING PROCESS: Implementation

4. When monitoring a patient's response to oral antidiabetic drugs, the nurse knows that which laboratory result would indicate a therapeutic response? a. Random blood glucose level 180 mg/dL b. Blood glucose level of 50 mg/dL after meals c. Fasting blood glucose level between 92 mg/dL d. Evening blood glucose level below 80 mg/dL

ANS: C The American Diabetes Association recommends a fasting blood glucose level of between 80 and 130 mg/dL for diabetic patients. The other options are incorrect. DIF: COGNITIVE LEVEL: Applying (Application) REF: p. 504 TOP: NURSING PROCESS: Evaluation

3. A patient is being treated for chemotherapy-induced nausea and vomiting (CINV) with ondansetron [Zofran] and dexamethasone. The patient reports getting relief during and immediately after chemotherapy but has significant nausea and vomiting several days after each chemotherapy treatment. What will the nurse do? a. Contact the provider to discuss increasing the dose of ondansetron. b. Suggest giving prolonged doses of dexamethasone. c. Suggest adding aprepitant [Emend] to the medication regimen. d. Tell the patient to ask the provider about changing the ondansetron to aprepitant.

ANS: C The current regimen of choice for patients taking highly emetogenic drugs consists of three agents: aprepitant plus dexamethasone plus a 5-HT3 antagonist, such as ondansetron. Aprepitant has a prolonged duration of action and can prevent delayed CINV as well as acute CINV. Increasing the dose of ondansetron will not help treat the delayed CINV. Glucocorticoids should be given intermittently and for short periods to avoid side effects. Changing the ondansetron to aprepitant is not recommended.

12. A patient will be taking bismuth subsalicylate (Pepto-Bismol) to control diarrhea. When reviewing the patient's other ordered medications, the nurse recognizes that which medication will interact significantly with the Pepto-Bismol? a. Acetaminophen (Tylenol), an analgesic b. Levothyroxine (Synthroid), a thyroid replacement drug c. Warfarin (Coumadin), an anticoagulant d. Fluoxetine (Prozac), an antidepressant

ANS: C The oral anticoagulant warfarin is more likely to cause increased bleeding times or bruising when co-administered with adsorbents. This is thought to be because the adsorbents bind to vitamin K, which is needed to make certain clotting factors. Vitamin K is synthesized by the normal bacterial flora in the bowel. The other options are incorrect. DIF: COGNITIVE LEVEL: Applying (Application) REF: p. 815 TOP: NURSING PROCESS: Implementation

9. When evaluating a patient's use of a metered-dose inhaler (MDI), the nurse notes that the patient is unable to coordinate the activation of the inhaler with her breathing. What intervention is most appropriate at this time? a. Notify the doctor that the patient is unable to use the MDI. b. Obtain an order for a peak flow meter. c. Obtain an order for a spacer device. d. Ask the prescriber if the medication can be given orally.

ANS: C The use of a spacer may be indicated with metered-dose inhalers, especially if success with inhalation is limited. The other options are not appropriate interventions. DIF: COGNITIVE LEVEL: Applying (Application) REF: p. 591 TOP: NURSING PROCESS: Implementation

4. The prescriber has changed the patient's medication regimen to include the leukotriene receptor antagonist (LTRA) montelukast (Singulair) to treat asthma. The nurse will emphasize which point about this medication? a. The proper technique for inhalation must be followed. b. The patient needs to keep it close by at all times to treat acute asthma attacks. c. It needs to be taken every day on a continuous schedule, even if symptoms improve. d. When the asthma symptoms improve, the dosage schedule can be tapered and eventually discontinued.

ANS: C These drugs are indicated for chronic, not acute, asthma and are to be taken every day on a continuous schedule, even if symptoms improve. These drugs are taken orally. DIF: COGNITIVE LEVEL: Applying (Application) REF: p. 592 TOP: NURSING PROCESS: Implementation

11. A nurse is admitting a patient to the hospital who reports having recurrent, crampy abdominal pain followed by diarrhea. The patient tells the nurse that the diarrhea usually relieves the pain and that these symptoms have occurred daily for the past 6 months. The patient undergoes a colonoscopy, for which the findings are normal. The nurse will plan to teach this patient to: a. use antispasmodic medications. b. avoid food containing lactose and gluten. c. keep a food, stress, and symptom diary. d. use antidiarrheal drugs to manage symptoms.

ANS: C This patient shows signs of irritable bowel syndrome (IBS), which can be managed with drug and nondrug therapies. Patients should be taught to keep a log to identify foods and stressors that trigger symptoms. Antispasmodic medications frequently are used, but there is no clear evidence of their benefit. Patients with malabsorption disorders may need to avoid lactose or gluten but only if indicated. Antidiarrheal drugs do not have clear benefits, even though they are commonly used.

A clinic patient who has cirrhosis of the liver develops hypervolemic hpernatremia. Which medication will the nurse expect the provider to order? a.Conivaptan [Vaprisol] b.Desmopressin [DDAVP] c.Tolvaptan [Samsca] d.Vasopressin [Pitressin]

ANS: C Tolvaptan is an oral vasopressin antagonist indicated for reducing hypernatremia in patients with euvolemic or hypervolemic hypernatremia. Conivaptan has the same indications, but it is administered IV for hospitalized patients. Desmopressin and vasopressin are antidiuretic hormone replacement medications that will make the condition worse.

6. A patient who has type 2 diabetes is scheduled for an oral endoscopy and has been NPO (nothing by mouth) since midnight. What is the best action by the nurse regarding the administration of her oral antidiabetic drugs? a. Administer half the original dose. b. Withhold all medications as ordered. c. Contact the prescriber for further orders. d. Give the medication with a sip of water.

ANS: C When the diabetic patient is NPO, the prescriber needs to be contacted for further orders regarding the administration of the oral antidiabetic drugs. The other options are incorrect. DIF: COGNITIVE LEVEL: Applying (Application) REF: p. 518 TOP: NURSING PROCESS: Implementation

1. A patient is about to receive a dose of octreotide (Sandostatin). The nurse will assess for which contraindications or cautions? (Select all that apply.) a. Carcinoid crisis b. Diarrhea c. Type 1 diabetes mellitus d. Gallbladder disease e. Chronic renal failure f. Esophageal varices

ANS: C, D, E Octreotide is used with caution in patients with diabetes (type 1 or 2), gallbladder impairment, or renal impairment. Carcinoid crisis, which may be associated with severe diarrhea and flushing, is an indication for octreotide; esophageal varices are also an indication. DIF: COGNITIVE LEVEL: Applying (Application) REF: p. 486 TOP: NURSING PROCESS: Assessment

5. A nursing student asks a nurse working in the newborn nursery how vitamin K deficiency is treated in newborns. Which response by the nurse is correct? a. "A newborn infant's intestinal flora will produce vitamin K." b. "If coagulation studies show a bleeding disorder, oral menadione is given." c. "Menadione is given intravenously shortly after delivery." d. "Phytonadione is given intramuscularly immediately after delivery."

ANS: D A normal infant is born vitamin K deficient, and all infants are given an IM injection of phytonadione immediately after delivery. Newborns have not developed the intestinal flora needed to produce sufficient amounts of vitamin K. Menadione is not used in infants because of the increased risk of hyperbilirubinemia associated with its use. PTS: 1 DIF: Cognitive Level: Application REF: p. 991

13. A patient asks how she can consume optimal amounts of folate, because she is trying to get pregnant. The nurse will advise this patient to do what? a. Eat foods naturally high in folate. b. Stop worrying, because folate is present in many foods. c. Take synthetic folate on an empty stomach. d. Take synthetic folate in addition to foods high in folate.

ANS: D All women of child-bearing age need 400 to 800 mg of synthetic folate in addition to dietary sources to help prevent neural tube defects in the fetus. The bioavailability of synthetic folate is increased in the presence of food. Dietary folate is not sufficient to prevent neural tube defects, even when these foods are consumed in increased amounts. Taking folate on an empty stomach reduces the amount absorbed. PTS: 1 DIF: Cognitive Level: Application REF: p. 994

12. A patient with type 1 diabetes who takes insulin reports taking propranolol for hypertension. Why is the nurse concerned? a. The beta blocker can cause insulin resistance. b. Using the two agents together increases the risk of ketoacidosis. c. Propranolol increases insulin requirements because of receptor blocking. d. The beta blocker can mask the symptoms of hypoglycemia.

ANS: D Beta blockers can delay awareness of and response to hypoglycemia by masking signs associated with stimulation of the sympathetic nervous system (eg, tachycardia, palpitations) that hypoglycemia normally causes. Furthermore, beta blockade impairs glycogenolysis, which is one means by which the body can counteract a fall in blood glucose; beta blockers, therefore, can worsen insulin-induced hypoglycemia. Propranolol does not cause insulin resistance. The incidence of DKA is not increased by concurrent use of propranolol and insulin. Insulin requirements are not increased because of receptor blocking by propranolol.

1. A patient, newly diagnosed with hypothyroidism, receives a prescription for a thyroid hormone replacement drug. The nurse assesses for which potential contraindication to this drug? a. Infection b. Diabetes mellitus c. Liver disease d. Recent myocardial infarction

ANS: D Contraindications to thyroid preparations include known drug allergy to a given drug product, recent myocardial infarction, adrenal insufficiency, and hyperthyroidism. The other options are incorrect. DIF: COGNITIVE LEVEL: Understanding (Comprehension) REF: p. 493 TOP: NURSING PROCESS: Assessment

4. The nurse is counseling a patient about calcium supplements. Which dietary information is appropriate during this teaching session? a. "Take oral calcium supplements with meals." b. "There are no drug interactions with calcium products." c. "Avoid foods that are high in calcium, such as beef, egg yolks, and liver." d. "Be sure to eat foods high in calcium, such as dairy products and salmon."

ANS: D Foods high in calcium include dairy products, fortified cereals, calcium-fortified orange juice, sardines, and salmon. Patients can be encouraged to add dietary sources of calcium to their diets. Oral-dosage forms of calcium need to be given 1 to 3 hours after meals. Calcium salts will bind with tetracycline and quinolone antibiotics and result in an insoluble complex. DIF: COGNITIVE LEVEL: Analyzing (Analysis) REF: p. 843 |pp. 857-858 TOP: NURSING PROCESS: Implementation

A child with Prader-Willi syndrome (PWS) has short stature, and the provider is considering treatment with growth hormone. Which aspect of this child's history should the nurse report to the provider? a.Behavior problems b.Low muscle tone c.Mental impairment d.Sleep apnea

ANS: D Growth hormone must be avoided in patients with PWS who are severely obese, who have a history of upper airway obstruction or sleep apnea, or who have severe respiratory impairment because of a risk of sudden death associated with these disorders. Behavior problems, low muscle tone, and mental impairment are commonly associated with PWS and are not contraindications to treatment with growth hormone

3. The nurse is teaching a group of patients about self-administration of insulin. What content is important to include? a. Patients need to use the injection site that is the most accessible. b. If two different insulins are ordered, they need to be given in separate injections. c. When mixing insulins, the cloudy (such as NPH) insulin is drawn up into the syringe first. d. When mixing insulins, the clear (such as regular) insulin is drawn up into the syringe first.

ANS: D If mixing insulins in one syringe, the clear (regular) insulin is always drawn up into the syringe first. Patients always need to rotate injection sites. Mixing of insulins may be ordered. DIF: COGNITIVE LEVEL: Applying (Application) REF: p. 516 TOP: NURSING PROCESS: Implementation

5. Which statement is accurate about the long-term complications of diabetes? a. Long-term complications are almost always the result of hypoglycemia and ketoacidosis. b. The complication rates for patients with optimally controlled type 2 diabetes are the same as for those whose disease is not optimally controlled. c. Optimal control of type 1 diabetes produces excessive episodes of life-threatening hypoglycemia. d. Optimal control of both types of diabetes reduces the risk of eye, kidney, and nerve damage.

ANS: D In both types of diabetes, optimal control of the disease slows the development of microvascular complications. Short-term complications are more apt to result from hypoglycemia and ketoacidosis. Patients with type 2 diabetes have fewer complications if their blood sugar level is optimally controlled. Hypoglycemia does not occur more frequently in patients with optimally controlled type 1 diabetes.

3. The order reads, "Give levothyroxine (Synthroid), 200 mg, PO once every morning." Which action by the nurse is correct? a. Give the medication as ordered. b. Change the dose to 200 mcg because that is what the prescriber meant. c. Hold the drug until the prescriber returns to see the patient. d. Question the order because the dose is higher than 200 mcg.

ANS: D Levothyroxine is dosed in micrograms. A common medication error is to write the intended dose in milligrams instead of micrograms. If not caught, this error would result in a thousandfold overdose. Doses higher than 200 mcg need to be questioned in case this error has occurred. The other options are incorrect. DIF: COGNITIVE LEVEL: Analyzing (Analysis) REF: p. 494 TOP: NURSING PROCESS: Planning

18. A patient who has traveler's diarrhea asks the nurse about using loperamide to stop the symptoms. What will the nurse tell the patient about this drug? a. "Loperamide is used for moderate to severe symptoms only." b. "This drug is useful as prophylaxis to prevent symptoms." c. "This drug is only effective to treat certain infectious agents." d. "Use of this drug may prolong symptoms by slowing peristalsis."

ANS: D Loperamide is a nonspecific antidiarrheal that slows peristalsis; by this action, it may delay transit of the causative organism and may prolong the infection. It is used to treat mild symptoms and is used once symptoms start. It is not an antibiotic agent and is nonspecific.

A 7-year-old child who is otherwise healthy is receiving mecasermin [Increlex] replacement therapy to treat severe primary deficiency of insulin-like growth factor-1 (IGF-1). The child develops tonsillar hypertrophy. The nurse anticipates that the provider will recommend: a.antibiotics. b.reducing the dose of mecasermin. c.discontinuing the mecasermin. d.tonsillectomy.

ANS: D Mecasermin can cause hypertrophy of the tonsils, which can be managed by tonsillectomy if needed. Antibiotics are not indicated, because the tonsil enlargement is not caused by infection. Neither reducing the dose of mecasermin nor discontinuing the drug is indicated.

17. A patient with gastroesophageal reflux disease (GERD) is to begin taking oral metoclopramide [Reglan]. The patient asks the nurse about the medication. Which response by the nurse is correct? a. "After 3 months, if the drug is not effective, you may need to increase the dose." b. "Metoclopramide may cause hiccups, especially after meals." c. "Serious side effects may occur but will stop when the drug is discontinued." d. "You should take the drug 30 minutes before each meal and at bedtime."

ANS: D Metoclopramide should be given 30 minutes before meals and at bedtime. Metoclopramide should not be used long term, and increasing the dose after 3 months of therapy is not recommended. Metoclopramide is used to treat hiccups; it does not cause hiccups. Tardive dyskinesia is a serious and irreversible side effect.

2. A patient with hypothyroidism is given a prescription for levothyroxine (Synthroid). When the nurse explains that this is a synthetic form of the thyroid hormone, he states that he prefers to receive more "natural" forms of drugs. What will the nurse explain to him about the advantages of levothyroxine? a. It has a stronger effect than the natural forms. b. Levothyroxine is less expensive than the natural forms. c. The synthetic form has fewer adverse effects on the gastrointestinal tract. d. The half-life of levothyroxine is long enough to permit once-daily dosing.

ANS: D One advantage of levothyroxine over the natural forms is that it can be administered only once a day because of its long half-life. The other options are incorrect. DIF: COGNITIVE LEVEL: Applying (Application) REF: p. 493 TOP: NURSING PROCESS: Implementation

7. When educating a patient recently placed on inhaled corticosteroids, the nurse will discuss which potential adverse effects? a. Fatigue and depression b. Anxiety and palpitations c. Headache and rapid heart rate d. Oral candidiasis and dry mouth

ANS: D Oral candidiasis and dry mouth are two possible adverse effects of inhaled corticosteroids. The other responses are incorrect. DIF: COGNITIVE LEVEL: Understanding (Comprehension) REF: p. 592 TOP: NURSING PROCESS: Planning

18. The nurse is teaching a review class to nurses about diabetes mellitus. Which statement by the nurse is correct? a. "Patients with type 2 diabetes will never need insulin." b. "Oral antidiabetic drugs are safe for use during pregnancy." c. "Pediatric patients cannot take insulin." d. "Insulin therapy is possible during pregnancy if managed carefully."

ANS: D Oral medications are generally not recommended for pregnant patients because of a lack of firm safety data. For this reason, insulin therapy is the only currently recommended drug therapy for pregnant women with diabetes. Insulin is given to pediatric patients, with extreme care. Patients with type 2 diabetes may require insulin in certain situations or as their disease progresses. DIF: COGNITIVE LEVEL: Applying (Application) REF: p. 506 TOP: NURSING PROCESS: Implementation

16. A nurse is caring for a patient with cancer who has been undergoing chemotherapy. The patient has oral mucositis as a result of the chemotherapy, and the provider has ordered palifermin [Kepivance]. Which is an appropriate nursing action when giving this drug? a. Administering the drug as a slow IV infusion b. Flushing the IV line with heparin before infusing the drug c. Giving the drug within 6 hours of the chemotherapy d. Warning the patient about the potential for distortion of taste

ANS: D Palifermin is generally well tolerated but commonly causes reactions involving the skin and mouth, including taste distortion. The drug should be given as an IV bolus. Palifermin binds with heparin, so the IV line should not be flushed with heparin before giving palifermin. If the interval between administration of palifermin and the chemotherapeutic drugs is too short, palifermin actually may increase the severity and duration of oral mucositis; therefore, palifermin should be given 24 hours before the chemotherapeutic drugs.

8. The nurse is teaching patients about self-injection of insulin. Which statement is true regarding injection sites? a. Avoid the abdomen because absorption there is irregular. b. Choose a different site at random for each injection. c. Give the injection in the same area each time. d. Rotate sites within the same location for about 1 week before rotating to a new location.

ANS: D Patients taking insulin injections need to be instructed to rotate sites, but to do so within the same location for about 1 week (so that all injections are rotated in one area—for example, the right arm—before rotating to a new location, such as the left arm). Also, each injection needs to be at least to 1 inch away from the previous site. DIF: COGNITIVE LEVEL: Applying (Application) REF: p. 517 TOP: NURSING PROCESS: Implementation

8. A nurse provides dietary counseling for a patient newly diagnosed with type 1 diabetes. Which instruction should be included? a. "You may eat any foods you want and cover the glucose increase with sliding scale, regular insulin." b. "Most of the calories you eat should be in the form of protein to promote fat breakdown and preserve muscle mass." c. "Your total caloric intake should not exceed 1800 calories in a 24-hour period." d. "You should use a carbohydrate counting approach to maintain glycemic control."

ANS: D Patients with diabetes should be given intensive insulin therapy education using either a carbohydrate counting or experience-based estimation approach in achieving glycemic control. A patient with diabetes cannot eat any foods desired and then cover the glucose increase with a sliding scale of regular insulin. Evidence suggests that there is not an ideal percentage of calories that should be ingested from carbohydrate, fat, or protein. Every patient with diabetes must be assessed individually to determine the number of total calories the person should have daily. The total caloric intake should be spread evenly throughout the day, with meals spaced 4 to 5 hours apart.

4. A patient is about to undergo a diagnostic bowel procedure. The nurse expects which drug to be used to induce total cleansing of the bowel? a. Docusate sodium (Colace) b. Lactulose (Enulose) c. Mineral oil d. Polyethylene glycol 3350 (GoLYTELY)

ANS: D Polyethylene glycol is a very potent laxative that induces total cleansing of the bowel and is most commonly used before diagnostic or surgical bowel procedures. The other options are incorrect. DIF: COGNITIVE LEVEL: Understanding (Comprehension) REF: p. 822 TOP: NURSING PROCESS: Planning

4. The parent of a child with cerebral palsy reports that the child has pebble-like stools most of the time and seems uncomfortable if several days have passed between stools. The nurse will suggest that the parent discuss which medication with the child's provider? a. Bisacodyl [Dulcolax] suppositories b. Magnesium citrate c. Methylcellulose [Citrucel] d. Polyethylene glycol [MiraLax]

ANS: D Polyethylene glycol is an osmotic laxative widely used for chronic constipation, which this child has, because it provides relief from abdominal discomfort, improves stool consistency, and increases frequency. Bisacodyl is not recommended for long-term use. Magnesium citrate causes increased water loss, and methylcellulose can also cause impaction.

11. A patient who has been on antibiotic therapy for 2 weeks has developed persistent diarrhea. The nurse expects which medication class to be ordered to treat this diarrhea? a. Lubricants b. Adsorbents c. Anticholinergics d. Probiotics

ANS: D Probiotics work by replenishing bacteria that may have been destroyed by antibiotic therapy, thus restoring the balance of normal flora and suppressing the growth of diarrhea-causing bacteria. DIF: COGNITIVE LEVEL: Applying (Application) REF: p. 815 TOP: NURSING PROCESS: Implementation

6. A patient has been taking psyllium [Metamucil] two to three times daily for several days. The patient complains of stomach pain but has not had a stool. What will the nurse do? a. Ask the patient to drink a full glass of water. b. Give another dose of the psyllium. c. Request an order for a bisacodyl [Dulcolax] suppository. d. Palpate the patient's abdomen and auscultate for bowel sounds.

ANS: D Psyllium is a bulk-forming laxative and can cause a fecal impaction or obstruction. Abdominal pain can be a sign of impaction or obstruction, so the nurse should assess the patient for this complication. Giving fluids by mouth, administering more laxative, or giving a suppository are all contraindicated if an obstruction has developed and should not be done until this condition is ruled out.

2. A patient with type 1 diabetes is eating breakfast at 7:30 AM. Blood sugars are on a sliding scale and are ordered before a meal and at bedtime. The patient's blood sugar level is 317 mg/dL. Which formulation of insulin should the nurse prepare to administer? a. No insulin should be administered. b. NPH c. 70/30 mix d. Lispro [Humalog]

ANS: D Regular insulin is indicated for sliding scale coverage. Insulin is definitely indicated for this high blood sugar level. NPH is used for scheduled insulin doses and is a longer-acting insulin. A 70/30 mix is also used for scheduled insulin coverage.

10. The nurse is providing instructions about the Advair inhaler (fluticasone propionate and salmeterol). Which statement about this inhaler is accurate? a. It is indicated for the treatment of acute bronchospasms. b. It needs to be used with a spacer for best results. c. Patients need to avoid drinking water for 1 hour after taking this drug. d. It is used for the prevention of bronchospasms.

ANS: D Salmeterol is a long-acting beta2 agonist bronchodilator, while fluticasone is a corticosteroid. In combination, they are used for the maintenance treatment of asthma and COPD. As a long-acting inhaler, Advair is not appropriate for treatment of acute bronchospasms. The other statements are incorrect. DIF: COGNITIVE LEVEL: Applying (Application) REF: p. 583 TOP: NURSING PROCESS: Implementation

9. A patient has been taking levothyroxine (Synthroid) for more than one decade for primary hypothyroidism. Today she calls because she has a cousin who can get her the same medication in a generic form from a pharmaceutical supply company. Which is the nurse's best advice? a. "This would be a great way to save money." b. "There's no difference in brands of this medication." c. "This should never be done; once you start with a certain brand, you must stay with it." d. "It's better not to switch brands unless we check with your doctor."

ANS: D Switching brands of levothyroxine during treatment can destabilize the course of treatment. Thyroid function test results need to be monitored more carefully when switching products. DIF: COGNITIVE LEVEL: Analyzing (Analysis) REF: p. 496 TOP: NURSING PROCESS: Implementation

13. A patient has been taking senna [Senokot] for several days, and the nurse notes that the urine is yellowish-brown. What does the nurse know about this symptom? a. It indicates that renal failure has occurred. b. It is caused by dehydration, which is a laxative side effect. c. It is a sign of toxicity, indicating immediate withdrawal of the drug. d. It is an expected, harmless effect of senna.

ANS: D Systemic absorption of senna, followed by renal excretion, may impart a harmless yellowish-brown or pink color to the urine. This symptom is not an indication of renal failure, dehydration, or toxicity.

20. When teaching a patient who is starting metformin (Glucophage), which instruction by the nurse is correct? a. "Take metformin if your blood glucose level is above 150 mg/dL." b. "Take this 60 minutes after breakfast." c. "Take the medication on an empty stomach 1 hour before meals." d. "Take the medication with food to reduce gastrointestinal (GI) effects."

ANS: D The GI adverse effects of metformin can be reduced by administering it with meals. The other options are incorrect. DIF: COGNITIVE LEVEL: Analyzing (Analysis) REF: p. 508 TOP: NURSING PROCESS: Implementation

9. What is the most reliable measure for assessing diabetes control over the preceding 3-month period? a. Self-monitoring blood glucose (SMBG) graph report b. Patient's report c. Fasting blood glucose level d. Glycosylated hemoglobin level

ANS: D The glycosylated hemoglobin level tells much about what the plasma glucose concentration has been, on average, over the previous 2 to 3 months. The SMBG graph report is done by the patient and indicates each blood sugar level the patient has on a daily basis. It is not as reliable as the glycosylated hemoglobin level, because the equipment used might not be accurate and the testing may not reflect actual measurements 100% of the time. The patient's report of blood sugar levels is not considered as accurate as the glycosylated hemoglobin level for the same reason that the SMBG is not. One fasting blood glucose level indicates the patient's blood sugar level for that one time when it was obtained.

1. A nurse is discussing nutrition with a patient who expresses concern about not always consuming vitamins in amounts consistent with the recommended dietary allowances (RDAs). What will the nurse tell this patient? a. The RDA is only an estimate of the amount of vitamins required and does not represent a scientific number. b. The RDA is the amount of vitamin needed by 50% of the population regardless of age or gender. c. The RDA represents the highest amount that can be consumed safely, so amounts less than this can be sufficient. d. The RDA represents an average, so low intake one day can be compensated for by increased amounts on another day.

ANS: D The recommended dietary allowance is the average daily dietary intake sufficient to meet the nutrient requirements of nearly all healthy individuals. Because it represents an average, a low amount one day can be compensated for by an increased amount on another day. The RDA values are determined through extensive experimental data and are not estimates. The estimated average requirement (EAR) is the level that meets the nutritional requirements of 50% of healthy individuals. The tolerable upper intake level (UL) is the highest average daily intake that can be consumed without risk of adverse effects. PTS: 1 DIF: Cognitive Level: Analysis REF: p. 987

1. A patient is taking intravenous aminophylline for a severe exacerbation of chronic obstructive pulmonary disease. The nurse will assess for which therapeutic response? a. Increased sputum production b. Increased heart rate c. Increased respiratory rate d. Increased ease of breathing

ANS: D The therapeutic effects of bronchodilating drugs such as xanthine derivatives include increased ease of breathing. The other responses are incorrect. DIF: COGNITIVE LEVEL: Applying (Application) REF: p. 593 TOP: NURSING PROCESS: Evaluation

11. A 19-year-old woman has been diagnosed with hypothyroidism and has started thyroid replacement therapy with levothyroxine (Synthroid). After 6 months, she calls the nurse to say that she feels better and wants to stop the medication. Which response by the nurse is correct? a. "You can stop the medication if your symptoms have improved." b. "You need to stay on the medication for at least 1 year before a decision about stopping it can be made." c. "You need to stay on this medication until you become pregnant." d. "Medication therapy for hypothyroidism is usually lifelong, and you should not stop taking the medication."

ANS: D These medications must never be abruptly discontinued, and lifelong therapy is usually the norm. The other options are incorrect. DIF: COGNITIVE LEVEL: Applying (Application) REF: p. 497 TOP: NURSING PROCESS: Assessment

8. A nurse is caring for a patient who recently immigrated from a Third World country. The patient is thin and appears malnourished. The nurse notes that the patient has loose and missing teeth, gingivitis, and bleeding gums. The patient has multiple sores and ecchymoses. The nurse will expect the provider to order: a. cyanocobalamin (vitamin B12). b. high-dose nicotinic acid. c. intramuscular thiamine for 1 to 2 weeks. d. intravenous ascorbic acid.

ANS: D This patient has scurvy with acute, severe symptoms and needs intravenous vitamin C. The patient does not have signs of vitamin B12 deficiency, niacin deficiency, or thiamine deficiency. PTS: 1 DIF: Cognitive Level: Application REF: p. 992

10. A patient is preparing to travel to perform missionary work in a region with poor drinking water. The provider gives the patient a prescription for ciprofloxacin [Cipro] to take on the trip. What will the nurse instruct this patient to do? a. Combine the antibiotic with an antidiarrheal medication, such as loperamide. b. Start taking the ciprofloxacin 1 week before traveling. c. Take 1 tablet of ciprofloxacin with each meal for best results. d. Use the drug if symptoms are severe or do not improve in a few days.

ANS: D Traveler's diarrhea is generally caused by Escherichia coli; treatment is usually unnecessary, because the disease runs its course in a few days. If symptoms are severe or prolonged, an antibiotic, such as ciprofloxacin, may be helpful. Patients should be instructed to take it only if needed. Antidiarrheal medications may just slow the export of the organism and prolong the course of the disease, but they may be used when symptoms are mild for relief from discomfort. Prophylactic treatment with antibiotics is not recommended. Ciprofloxacin is given twice daily, not with meals.

2. A pregnant patient is discussing nutrition and vitamin supplements with the nurse. Which statement by the patient indicates an understanding of the use of nutrition and supplements during pregnancy? a. "I can get adequate folic acid by consuming foods fortified with synthetic folate." b. "I need reduced amounts of vitamin C while pregnant to lower my risk of hemorrhage." c. "I should take vitamin K so my baby won't be vitamin K deficient at birth." d. "Excessive amounts of vitamin A [retinol] can cause birth defects in my baby."

ANS: D Vitamin A in high doses can cause birth defects, so pregnant women should be cautioned about exceeding the UL and probably the RDA for vitamin A while pregnant. Pregnant women need to take 400 to 800 mg of supplemental folic acid in addition to that in food. Vitamin C deficiency, not excess, can lead to bleeding disorders. Maternal intake of vitamin K will not prevent infants from being born vitamin K deficient. PTS: 1 DIF: Cognitive Level: Application REF: pp. 988-989 | pp. 991-992 | p. 994

3. A child is diagnosed with rickets. The nurse knows that this child is most likely deficient in which vitamin? a. Niacin (nicotinic acid) b. Thiamin (vitamin B1) c. Vitamin C (ascorbic acid) d. Vitamin D

ANS: D Vitamin D plays a critical role in the regulation of the metabolism of calcium and phosphorus. Classical effects of deficiency are rickets in children and osteomalacia in adults. Niacin, thiamin, and vitamin C do not play a role in the prevention of rickets. PTS: 1 DIF: Cognitive Level: Analysis REF: p. 990

The nurse is caring for a patient receiving desmopressin [Stimate]. The nurse is performing a physical assessment and notes that the patient is drowsy and listless and complains of a recent onset of headache. Which finding would be most consistent with these symptoms? a.Hyperglycemia b.Hypernatremia c.Hypertension d.Water intoxication

ANS: D Water intoxication is manifested by drowsiness, listlessness, and headache. The patient's signs and symptoms are not consistent with hypernatremia or hyperglycemia, and there is no indication of hypertension.

9. A patient accidentally took an overdose of the anticoagulant warfarin (Coumadin), and the nurse is preparing to administer vitamin K as an antidote. Which statement about vitamin K is accurate? a. The vitamin K dose will be given intramuscularly. b. The patient will take oral doses of vitamin K after the initial injection. c. The vitamin K cannot be given if the patient has renal disease. d. The patient will be unresponsive to warfarin therapy for 1 week after the vitamin K is given.

ANS: D When vitamin K is used as an antidote to warfarin therapy, the patient becomes unresponsive to warfarin for approximately 1 week after vitamin K administration. The use of vitamin K products is contraindicated in patients who are in the last few weeks of pregnancy and in patients with severe hepatic disease. Vitamin K is given subcutaneously and not intramuscularly when used to reverse warfarin effects. DIF: COGNITIVE LEVEL: Applying (Application) REF: p. 847 TOP: NURSING PROCESS: Implementation

Textbook Case Studies A 50-year-old attorney has self-treated for heartburn for years by drinking large amounts of antacids. She finally made an appointment with her family practice physician, who referred her to a gastroenterologist. Her family practice physician instructed her to stop taking the antacids. In a few weeks, the attorney had an endoscopy, and it was discovered that she had gastroesophgeal reflux disease (GERD) and gastritis secondary to stress-induced hyper-acidity. The gastoenterologist has prescribed the proton pump inhibitor (PPI) omeprazole (Prilosec) 20 mg once a day. 4. What patient teaching is important regarding the PPI?

Administer omeprazole before meals, and take the capsule whole and not crushed, opened, or chewed. It needs to be taken on an empty stomach, 30 to 60 minutes before breakfast, with a full glass of water.

ANS: A Sometimes H2 receptor antagonists such as cimetidine may cause adverse effects related to the central nervous system in the elderly, including confusion and disorientation. The nurse needs to be alert for mental status changes when giving these drugs, especially if the changes are new to the patient.

An elderly patient had gastric surgery due to a gastrointestinal bleed 3 days ago, and he has been stable since the surgery. This evening, his daughter tells the nurse, "He seems to be more confused this afternoon. He's never been like this. What could be the problem?" The nurse reviews the patient's medication record and suspects that which drug could be the cause of the patient's confusion? a.cimetidine (Tagamet) b.pantoprazole (Protonix) c.clarithromycin (Biaxin) d.sucralfate (Carafate)

Textbook Case Studies A 50-year-old attorney has self-treated for heartburn for years by drinking large amounts of antacids. She finally made an appointment with her family practice physician, who referred her to a gastroenterologist. Her family practice physician instructed her to stop taking the antacids. 1. Why did physician ask her to stop taking the antacids?

Antacids have been found to have many side effects, such as constipation and diarrhea, and may also lead to rebound of hyperacidity, which may indeed increase the detrimental effects of acid on the refulx disease and on any ulcerated areas in the stomach. Antacids elevate gastric pH, Which is why they have been used in the past; however, antacids do not affect the gastric production of acid, which the other drugs (such as H2-blocking drugs) do so effectively. In addition, long-term self-treatment with antacids may actually delay needed therapy for certain conditions, such as Helicobacter pylori, or more serious conditions, such as gastric ulcers.

ANS: C Medications are not to be taken, unless prescribed, within 1 to 2 hours of taking an antacid because of their impact on the absorption of many medications in the stomach.

At 0900, the nurse is about to give morning medications, and the patient has asked for a dose of antacid for severe heartburn. Which schedule for the antacid and medications is correct? a.Give both the antacid and medications at 0900. b.Give the antacid at 0900, and then the medications at 0930. c.Give the medications at 0900, and then the antacid at 1000. d.Give the medications at 0900, and then the antacid at 0915.

14. Under secretion of the adrenocortical hormones leads to: A. Cushing's syndrome B. Addison's disease

B

2. When discussing glucocorticoids to a patient, what statement by the nurse is accurate regarding the action of these medications? A. They decrease serum sodium and glucose levels. B. They regulate carbohydrate, fat, and protein metabolism. C. They stimulate defense mechanisms to produce immunity. D. They are produced in lower amounts during times of stress.

B

22. MOA for Glucocorticoids: A. Exerts effect by modifying enzyme activity B. Inhibit inflammatory and immune responses

B

25. Glucocorticoids are administered nasally for: A. Control of steroid-responsive bronchospastic states B. Rhinitis and to prevent the recurrence of polyps after surgical removal C. Inflammations of the eye, ear, and skin D. Shock, status asthmaticus and spinal cord injury

B

31. Adverse effects for adrenal drugs of this system include: - Convulsions, headache, vertigo, mood swings, nervousness, insomnia, "steroid psychosis" A. Cardiovascular B. CNS C. Endocrine D. GI E. Integumentary F. Musculoskeletal G. Ocular H. Other

B

5. Discharge teaching for a patient receiving glucocorticoids would include the use of which medication for pain management? A. Aspirin (Acetylsalicylic acid) B. Acetaminophen (Tylenol) C. Ibuprofen D. Naprosyn (Naproxen)

B

10. Which conditions is aminoglutethimide (Cytadren) used to treat? (Select all that apply.) A. Thyroid cancer B. Adrenal cancer C. Testicular cancer D. Cushing's syndrome E. Metastatic breast cancer

B, D, E

Which statement should the nurse include in the teaching for a patient who is to be started on zileuton [Zyflo]? A. "Use your zileuton [Zyflo] inhaler every 12 hours." B. "Have your blood drawn once a month for the next 3 months so that your liver function can be checked." C. "Take the zileuton [Zyflo] 2 hours before eating breakfast each day." D. "Take an extra dose of zileuton [Zyflo] if you have an asthmatic attack."

B. "Have your blood drawn once a month for the next 3 months so that your liver function can be checked." Zileuton can injure the liver, as evidenced by increased plasma levels of alanine aminotransferase (ALT). Therefore, ALT should be monitored once a month for 3 months, then every 2 to 3 months for the remainder of the first year, and periodically thereafter. Zileuton is an oral, not an inhaled, medication that is rapidly absorbed with or without food. The timing of administration with regard to meals is insignificant. Zileuton is used for asthma prophylaxis and cannot be used to abort an attack.

A nurse is teaching a patient about montelukast [Singulair]. Which statement by the patient would indicate that the nurse's teaching was effective? A. "I'll take a dose as soon as I feel short of breath and start to cough." B. "While taking this medicine, I may be able to reduce my steroid medication." C. "This is the priority medication for preventing exercise-induced asthma symptoms." D. "If I have nosebleeds or excessive bruising, I'll stop the medication immediately."

B. "While taking this medicine, I may be able to reduce my steroid medication." Montelukast is an antileukotriene agent. Combining montelukast with an inhaled glucocorticoid medication can improve asthma symptoms and thus may allow a reduction in the glucocorticoid dosage. The effects of montelukast develop slowly, so it cannot be used as a quick-relief agent. Short-acting beta2 agonists are preferred for exercise-induced asthma. Montelukast does not affect coagulation, so bleeding and bruising do not occur.

The nurse would question an order for somatrem (Protropin) in a patient with which condition? A. Dwarfism B. Acromegaly C. Growth failure D. Hypopituitarism

B. Acromegaly Somatrem is a synthetic form of growth hormone. Acromegaly is caused by excessive growth hormone, and thus this drug would be contraindicated.

To achieve therapeutic effectiveness, a nurse teaches a patient with chronic asthma to use an inhaled glucocorticoid medication according to which schedule? A. Only in an emergency B. On a continuing, daily basis C. To abort an asthma attack D. 2 weeks on, 2 weeks off

B. On a continuing, daily basis Glucocorticoid medications are the first-line therapy for asthma to reduce symptoms of inflammation. They should be taken for prophylaxis on a daily basis. Therapeutic effects develop slowly, so these drugs cannot be taken to abort an asthma attack or in an emergency. They are most effective when administered on a fixed schedule, not PRN.

A history of allergy to which substance is a contraindication to the use of ipratropium/albuterol [Combivent]? A. Mold B. Peanuts C. Penicillin D. Dairy products

B. Peanuts Patients with peanut allergy should avoid Combivent, which contains soya lecithin as a carrier. Soya is in the same plant family as peanuts, and about 10% of people with peanut allergy are cross-allergic to soya. Allergy to the remainder of the options is not a contraindication.

A nurse teaches a patient with chronic obstructive pulmonary disease (COPD) about the adverse effects of tiotropium [Spiriva]. Which behavior by the patient would indicate that the teaching has been effective? A. Combines tiotropium with an antacid B. Sucks on hard candy as needed C. Prevents constipation with a stool softener D. Wears long sleeves and a wide-brim hat

B. Sucks on hard candy as needed Tiotropium is an anticholinergic medication used to relieve bronchospasm associated with COPD. The most common adverse effect is dry mouth, and patients can suck on hard candy for symptomatic relief. It is not necessary to take an antacid medication, use a daily stool softener, or wear protective clothing when taking tiotropium.

Which drugs have an action similar to that of the naturally occurring hormone ADH? (Select all that apply) A. cosyntropin (Cortrosyn) B. desmopressin (DDAVP) C. somatropin (Humatrope) D. vasopressin (Pitressin) E. octreotide (Sandostatin)

B. desmopressin (DDAVP) D. vasopressin (Pitressin)

26. Glucocorticoids are administered topically for: A. Control of steroid-responsive bronchospastic states B. Rhinitis and to prevent the recurrence of polyps after surgical removal C. Inflammations of the eye, ear, and skin D. Shock, status asthmaticus and spinal cord injury

C

28. Contraindictions for adrenal drugs: A. Drug allergies B. Serious infections, including septicemia, systemic fungal infections, and varicella C. All of the above

C

3. The nurse would question a prescription for steroids in a patient with which condition? A. Asthma B. Spinal cord injury C. Diabetes mellitus D. Rheumatoid arthritis

C

32. Adverse effects for adrenal drugs of this system include: - Growth suppression, Cushing's syndrome, menstrual irregularities, carbohydrate intolerance, hyperglycemia A. Cardiovascular B. CNS C. Endocrine D. GI E. Integumentary F. Musculoskeletal G. Ocular H. Other

C

8. The nurse has a prescription for a patient to receive prednisone (Deltasone) to treat contact dermatitis. The nurse would question this prescription for this patient with what condition? A. Asthma B. Multiple sclerosis C. Acquired immune deficiency syndrome (AIDS) D. Chronic obstructive pulmonary disease

C

12. The adrenal cortex secretes the following corticosteroids: A. Epinephrine B. Norepinephrine C. Glucocorticoids D. Mineralocorticoids (primarily aldosterone)

C, D

A patient with asthma is scheduled to start taking a glucocorticoid medication with a metered-dose inhaler (MDI). The nurse should give the patient which instruction about correct use of the inhaler? A. "After you inhale the medication once, repeat until you obtain symptomatic relief." B. "Wait no longer than 30 seconds after the first puff before taking the second one." C. "Use a spacer with the inhaler and rinse your mouth after each dose administration." D. "Breathe in through the nose and hold for 2 seconds just before activating the inhaler."

C. "Use a spacer with the inhaler and rinse your mouth after each dose administration." Spacers are available for use with MDIs to prevent the patient from swallowing the dose and to allow for maximum delivery of medication to the lungs. Rinsing the mouth after administration is important for inhaled glucocorticoids to prevent candidiasis. Glucocorticoid inhalers are used for long-term prophylaxis of asthma, not for symptomatic relief. When two puffs are needed, an interval of at least 1 minute should separate the first puff from the second. Inhaling through the mouth just before activating the MDI is the proper technique.

Which is a priority nursing diagnosis for a patient receiving desmopressin (DDAVP)? A. Risk for injury B. Acute pain C. Excess fluid volume D. Deficient knowledge regarding medication

C. Excess fluid volume Desmopressin is a form of antidiuretic hormone, which increases sodium and water retention, leading to an alteration in fluid volume. Although the other nursing diagnoses may be appropriate, they are not a priority using Maslow's hierarchy of needs.

A patient who takes cromolyn for exercise-induced bronchospasm should follow which approach for maximum therapeutic effectiveness? A. It should be used infrequently because of systemic adverse effects. B. One 10-mg tablet should be taken at least 2 hours before exercising. C. It should be administered by inhalation 15 minutes before anticipated exertion. D. It should be used as a quick-relief agent if exercise triggers asthma symptoms.

C. It should be administered by inhalation 15 minutes before anticipated exertion. Cromolyn suppresses inflammation through inhibition of histamine release. It must be administered at least 15 minutes before exertion to prevent exercise-induced bronchospasm. It is administered by inhalation only. It is not a bronchodilator and cannot abort an ongoing attack. It is one of the safest antiasthma medications and has no systemic adverse effects.

A patient who is about to be given octreotide is also taking a diuretic, IV heparin, ciprofloxacin (Cipro), and an opioid as need for pain. The nurse will monitor for what possible interaction? A. Hypokalemia due to an interaction with the diuretic B. Decreased anticoagulation due to an interaction with the heparin C. Prolongation of the QT internal due to an interaction with ciprofloxacin D. Increased sedation if the opioid is given

C. Prolongation of the QT internal due to an interaction with ciprofloxacin

Which outcome should a nurse establish as a priority for a patient taking an oral glucocorticoid for long-term treatment of asthma? A. Increases the daily intake of vitamin D and calcium B. Records daily peak expiratory flow rates C. Supplements additional doses at times of stress D. Uses alternate-day therapy to reduce adverse effects

C. Supplements additional doses at times of stress Adrenal suppression can be profound with oral glucocorticoid use. It is a priority that patients take supplemental oral or intravenous doses at times of stress; failure to do so can be fatal. Alternate-day dosing, using a peak flowmeter, and minimizing bone loss with vitamin D and calcium intake are important; however, they are not as important as supplemental doses of glucocorticoid at times of stress.

1. Based on the nurse's knowledge of glucocorticoids, what instructions should be given for this drug? A. Take the medication every evening. B. Inform the patient that the drug can be taken with coffee. C. Advise that the drug does not have to be tapered before stopped. D. Advise to take drug with milk, other dairy products, or food.

D

16. Mineralocorticoids are: A. Topical B. Inhaled C. Nasal D. Systemic E. All of the above

D

17. Anti-adrenals, adrenal steroid inhibitors, are: A. Topical B. Inhaled C. Nasal D. Systemic E. All of the above

D

27. Glucocorticoids are administered parenteral for: A. Control of steroid-responsive bronchospastic states B. Rhinitis and to prevent the recurrence of polyps after surgical removal C. Inflammations of the eye, ear, and skin D. Shock, status asthmaticus and spinal cord injury

D

29. Use adrenal drugs with caution for patients with: A. Gastritis, reflux disease, ulcer disease B. Diabetes C. Cardiac/renal/liver dysfunction D. All of the above

D

33. Adverse effects for adrenal drugs of this system include: - Peptic ulcers with possible perforation, pancreatitis, abdominal distention A. Cardiovascular B. CNS C. Endocrine D. GI E. Integumentary F. Musculoskeletal G. Ocular H. Other

D

9. A pediatric patient has been prescribed methylprednisolone (Solu-Medrol) for an allergic response to food. The dose prescribed is 0.5 mg/kg IV every 6 hours. The patient's weight is 11.4 kg. How much will the nurse administer for one dose? A. 4.3 mg B. 3.1 mg C. 7.5 mg D. 5.7 mg

D

The nurse is reviewing the medication list for a patient who will be starting therapy with somatropin. Which type of drug would raise a concern that needs to be addressed before the patient starts the somatropin? A. Nonsteroidal anti-inflammatory drug for arthritis B. Antidepressant drug C. Penicillin D. Glucocorticoid

D. Glucocorticoid

A patient is taking oral theophylline for maintenance therapy of stable asthma. A nurse instructs the patient to avoid using which substance to prevent a complication? A. Echinacea B. Cimetidine [Tagamet] C. Sunscreen products D. Caffeine

D. Caffeine Theophylline is a methylxanthine that provides benefits through bronchodilation. It is used to reduce the frequency and severity of asthma attacks, especially those occurring at night. Caffeine also is a methylxanthine, and its pharmacologic actions may intensify the adverse effects of theophylline on the central nervous system (CNS) and heart. Sources of caffeine should be avoided. It is not necessary to avoid taking cimetidine or echinacea or using sunscreen products while taking theophylline.

When monitoring for the therapeutic effects of intranasal desmopressin (DDAVP) in a patient who has diabetes insidious, which assessment finding will the nurse look for as an indication that the medication therapy is successful? A. Increased insulin levels B. Decreased diarrhea C. Improved nasal patency D. Decreased thirst

D. Decreased thirst

A patient is admitted to the emergency department with acute severe exacerbation of asthma. Which drug should the nurse anticipate will be included in the treatment plan? A. Oral theophylline [Elixophyllin] B. Subcutaneous omalizumab [Xolair] C. Inhaled mometasone furoate [Asmanex] D. High-dose albuterol [Proventil] via nebulizer treatment

D. High-dose albuterol [Proventil] via nebulizer treatment Nebulized high-dose SABAs, such as albuterol, are administered to relieve airflow obstruction. Oral theophylline is used for maintenance therapy of chronic stable asthma, not for treatment of exacerbation. Omalizumab is a second-line agent indicated for allergy-related asthma and only when preferred options have failed. Inhaled glucocorticoids, such as mometasone furoate, are not used to abort an acute attack. During an exacerbation they are administered systemically.

After administering somatropin (Serostim), the nurse would assess for adverse effects by monitoring which parameters? A. Serum potassium levels B. Mental status C. Respiratory rate D. Serum glucose levels

D. Serum glucose levels Hyperglycemia and hypoglycemia are potential adverse effects of somatropin therapy.

A patient is suspected of having adrenocortical insufficiency. The nurse expects to administer which drug to aid in the diagnosis of this condition? A. octreotide (Sandostatin) B. vasopressin (Pitressin) C. somatropin (Humatrope) D. cosyntropin (Cortrosyn)

D. cosyntropin (Cortrosyn)

The nurse admitting a patient with acromegaly anticipates administering which medication? A. desmopressin (DDAVP) B. corticotropin (Acthar) C. somatropin (Nutropin) D. octreotide (Sandostatin)

D. octreotide (Sandostatin) Octreotide suppresses growth hormone, the culprit of acromegaly.

ANS: D Long-term self-medication with antacids may mask symptoms of serious underlying diseases, such as bleeding ulcer or malignancy. Patients with ongoing symptoms need to undergo regular medical evaluations, because additional medications or other interventions may be needed.

During an admission assessment, the patient tells the nurse that he has been self-treating his heartburn for 1 year with over-the-counter Prilosec OTC (omeprazole, a proton pump inhibitor). The nurse is aware that this self-treatment may have which result? a.No serious consequences b.Prevention of more serious problems, such as an ulcer c.Chronic constipation d.Masked symptoms of serious underlying diseases

15. Glucocorticoids are: A. Topical B. Inhaled C. Nasal D. Systemic E. All of the above

E

34. Adverse effects for adrenal drugs of this system include: - Fragile skin, petechiae, ecchymosis, facial erythema, poor wound healing, hirsutism, urticaria A. Cardiovascular B. CNS C. Endocrine D. GI E. Integumentary F. Musculoskeletal G. Ocular H. Other

E

Textbook Case Studies A 50-year-old attorney has self-treated for heartburn for years by drinking large amounts of antacids. She finally made an appointment with her family practice physician, who referred her to a gastroenterologist. Her family practice physician instructed her to stop taking the antacids. In a few weeks, the attorney had an endoscopy, and it was discovered that she had gastroesophgeal reflux disease (GERD) and gastritis secondary to stress-induced hyper-acidity. The gastoenterologist has prescribed the proton pump inhibitor (PPI) omeprazole (Prilosec) 20 mg once a day. 2. What other conditions will the gastroenterologist test for during this diagnostic stage?

Evaluate baseline renal and liver function. In addition, the gastroenterologist will test for the presence of H. pylori antibodies.

35. Adverse effects for adrenal drugs of this system include: - Muscle weakness, loss of muscle mass, osteoporosis A. Cardiovascular B. CNS C. Endocrine D. GI E. Integumentary F. Musculoskeletal G. Ocular H. Other

F

20. Anti-adrenals, adrenal steroid inhibitors, include: A. Beclomethasone (several formulations) B. Fluticasone propionate C. Hydrocortisone (several formulations) D. Cortisone E. Methylprednisolone F. Prednisone G. Fludrocortisone acetate H. Aminoglutethimide

H

37. Adverse effects for adrenal drugs of this system include: - Weight gain A. Cardiovascular B. CNS C. Endocrine D. GI E. Integumentary F. Musculoskeletal G. Ocular H. Other

H

23. Indications for adrenal drugs: A. Adrenocortical deficiency B. Collagen diseases C. Dermatologic diseases D. GI diseases E. Exacerbations of chronic respiratory illnesses, such as asthma and COPD F. Organ transplant (decrease immune response) G. Palliative management of leukemias and lymphomas H. Spinal cord injury I. All of the above

I

Preferred thyroid drug

Levothyroxine because its hormonal content is standardized and its effect is predictable

Textbook Case Studies A 50-year-old attorney has self-treated for heartburn for years by drinking large amounts of antacids. She finally made an appointment with her family practice physician, who referred her to a gastroenterologist. Her family practice physician instructed her to stop taking the antacids. In a few weeks, the attorney had an endoscopy, and it was discovered that she had gastroesophgeal reflux disease (GERD) and gastritis secondary to stress-induced hyper-acidity. The gastoenterologist has prescribed the proton pump inhibitor (PPI) omeprazole (Prilosec) 20 mg once a day. 3. What is the rational for use of the PPIs to treat GERD?

Proton pump inhibitors (PPIs) are used as long-term therapy to promote and maintain the healing of gastroesophageal reflux disease (GERD) and other hypersecretory disorders. The PPI inhibits the production of hydrochloric acid in the stomach.

ANS: B, C, E Antacids neutralize acid in the stomach. Magnesium-based antacids cause diarrhea, and aluminum-based antacids cause constipation. Calcium-based antacids often cause rebound hyperacidity.

The nurse is providing patient teaching about antacids. Which statements about antacids are accurate? (Select all that apply.) a.Antacids reduce the production of acid in the stomach. b.Antacids neutralize acid in the stomach. c.Rebound hyperacidity may occur with calcium-based antacids. d.Aluminum-based antacids cause diarrhea. e.Magnesium-based antacids cause diarrhea.

ANS: D All H2 receptor antagonists may inhibit the absorption of certain drugs, such as the antifungal ketoconazole, which require an acidic gastrointestinal environment for gastric absorption. The other options are incorrect.

The nurse is reviewing the medication orders for a patient who will be taking an H2 antagonist. Which drug may have an interaction if taken along with the H2 antagonist? a.ibuprofen (Motrin) b.ranitidine (Zantac) c.tetracycline (Doryx) d.ketoconazole (Nizoral)

ANS: D New concerns have arisen over the potential for long-term users of proton pump inhibitors (PPIs) to develop osteoporosis. This is thought to be due to the inhibition of stomach acid, and it is speculated that PPIs speed up bone mineral loss. The other options are incorrect.

The nurse is teaching a patient who will be taking a proton pump inhibitor as long-term therapy about potential adverse effects. Which statement is correct? a.Proton pump inhibitors can cause diarrhea. b.These drugs can cause nausea and anorexia. c.Proton pump inhibitors cause drowsiness. d.Long-term use of these drugs may contribute to osteoporosis.

Textbook Critical Thinking and Prioritization Questions 1. A patient with a history of decreased renal function tells the nurse, "I have finally found an antacid that gives me great relief!" The nurse checks the antacid's content and finds that the antacid is a combination of aluminum hydroxide and magnesium hydroxide. What is the nurse's priority action at this time? Explain your answer.

The nurse's priority action is to educate the patient about which antacids are safe for patients with decreased renal function. Both calcium-and magnesium-based antacids are more likely to accumulate to toxic levels in patients with renal disease. Therefore, if antacids are used in renal patients, preparations that are only aluminum-based should be chosen because they are generally more easily excreted than other categories of antacid. Sodium-based antacids may cause fluid retention and metabolic changes, so they also must be avoided by patients with renal problems.

Textbook Critical Thinking and Prioritization Questions 2. A patient tells the nurse, "I like taking antacids because they coat my stomach and protect my ulcer." What is the nurse's priority when answering the patient's question?

The priority is to explain to the patient how antacids work and how they are to be used for short-term relief only. Antacids do not coat the stomach. They elevate the gastric pH so that an acidic environment, which could cause more problems for patients with ulcers, does not exist. However, long-term use of antacids may mask symptoms of serious problems, such as gastric ulcers. In addition, overuse of antacids may cause acid rebound. Patients with chronic heartburn problems need to be checked thoroughly for the presence of more serious conditions.

ANS: B Both calcium- and magnesium-based antacids are more likely to accumulate to toxic levels in patients with renal disease and are commonly avoided in this patient group. The other options are incorrect.

When reviewing the health history of a patient who will be receiving antacids, the nurse recalls that antacids containing magnesium need to be used cautiously in patients with which condition? a.Peptic ulcer disease b.Renal failure c.Hypertension d.Heart failure

Preparation for the NCLEX® Examination Questions 2. The nurse will monitor a client taking an aluminum-containing antacid, such as aluminum hydroxide (Amphojel), for which adverse effect? a. Constipation b. Gastrointestinal (GI) upset c. Fluid retention d. Diarrhea

a Constipation Aluminum-and calcium-containing antacids cause constipation, magnesium-containing antacids cause diarrhea, and sodium-containing antacids cause sodium and fluid retention.

NCLEX EXAMINATION REVIEW QUESTIONS 5. A patient who is taking oral tetracycline complains of heartburn and requests an antacid. Which action by the nurse correct? a. Give the tetracycline, but delay the antacid for 1 to 2 hours. b. Give the antacid, but delay the tetracycline for at least 4 hours. c. Administer both medications together. d. Explain that the antacids cannot be given while the patient is taking the tetracycline.

a Give the tetracycline, but delay the antacids for 1 to 2 hours.

Preparation for the NCLEX® Examination Questions 1. The nurse will teach clients that antacids are effective in the treatment of hyperacidity based on which mechanism of action? a. Neutralizes gastric acid b. Decreases gastric pH c. Decreases stomach motility d. Decreases duodenal pH

a Neutralizing gastric acid Antacids work by neutralizing gastric acid, which would cause an increase in pH. They do not affect gastric motility.

NCLEX EXAMINATION REVIEW QUESTIONS 6. When the nurse is administering a proton pump inhibitor (PPI), which actions by the nurse are correct? (Select all that apply.) a. Giving the PPI on an empty stomach b. Giving the PPI with meals c. Making sure the patient dose not crush or chew the capsules d. Instructing the patient to open the capsule and chew the contents for best absorption e. Administering the PPI only when the patient complains of heartburn

a, c Give the PPI on an empty stomach, making sure the patient dose not crush or chew the capsules

A patient has been taking levothyroxine for several years and reports that "for the past 2 weeks, the drug doesn't seem to work as well as before." What will the nurse do? a. Ask the patient when the prescription was last refilled. b. Expect the patient to have an elevated temperature and tachycardia. c. Suggest that the patient begin taking calcium supplements. d. Tell the patient to try taking the medication with food.

a. Ask the patient when the prescription was last refilled. Not all levothyroxine preparations have the same drug bioavailability; therefore, if a patient is experiencing differing effects, the pharmacist may have switched brands. Asking a patient about a recent refill may help to explain why the drug has different effects. An elevated temperature and tachycardia would be signs of toxicity, not of a decrease in effectiveness. Calcium supplements and food would only interfere with absorption and further reduce the drug's effectiveness.

A patient with arthritis is admitted to the hospital. The patient's serum glucose level is 350 gm/dL, and the blood pressure is 182/98 mm Hg. The nurse notes that the patient's face appears rounded and puffy. The patient complains of feeling weak. What will the nurse do? a. Ask which drugs the patient takes for arthritis. b. Contact the provider to discuss whether the patient has a pituitary carcinoma. c. Request an order for ketoconazole [Nizoral]. d. Suspect that this patient has Addison's disease.

a. Ask which drugs the patient takes for arthritis. Many patients with arthritis are treated with glucocorticoids. Because the doses necessary to suppress inflammation are larger than the physiologic doses used to treat adrenal insufficiency, patients can develop signs of cortisol excess with cushingoid symptoms. This patient has an elevated glucose level, hypertension, and the characteristic moon facies and muscle weakness of Cushing's syndrome; therefore, the nurse would be correct to ask about the medications the patient takes for arthritis. A pituitary carcinoma could be the cause but is less likely. Ketoconazole is used after chemotherapy and radiation therapy in the treatment of a pituitary adenoma. This patient does not show signs of Addison's disease.

A patient is admitted with nausea, vomiting, diarrhea, and abdominal pain. The patient appears emaciated and complains of feeling weak. The nurse notes a heart rate of 98 beats per minute and a blood pressure of 88/54 mm Hg. The nurse reviews the chart and notes an increased serum potassium level and a decreased serum sodium level. The nurse expects the provider to order which medication initially? a. Cosyntropin [Cortrosyn] b. Dexamethasone c. Fludrocortisone [Florinef] d. Hydrocortisone

a. Cosyntropin [Cortrosyn] Cosyntropin is used to diagnose adrenal insufficiency, which this patient shows signs of having. Dexamethasone is used to diagnose Cushing's syndrome. Fludrocortisone, a mineralocorticoid, is used with hydrocortisone to treat primary adrenal insufficiency; both of these drugs will be administered after the diagnosis has been confirmed.

A 1-year-old child with cretinism has been receiving 8 mcg/kg/day of levothyroxine [Synthroid]. The child comes to the clinic for a well-child checkup. The nurse will expect the provider to: a. change the dose of levothyroxine to 6 mcg/kg/day. b. discontinue the drug if the child's physical and mental development are normal. c. increase the dose to accommodate the child's increased growth. d. stop the drug for 4 weeks and check the child's TSH level.

a. change the dose of levothyroxine to 6 mcg/kg/day. In the treatment of cretinism, thyroid dosing decreases with age. For infants 6 to 12 months of age, the dose is 6 mcg/kg/day. At 1 year of age, the dose is reduced to 5 to 6 mcg/kg/day. For all children, treatment should continue for 3 years. It is incorrect to increase the dose with age. After 3 years of therapy, the patient undergoes a trial of 4 weeks without the drug, followed by assessment of the TSH and T4 levels, to determine whether the drug may be discontinued.

When teaching a patient who has a new prescription for thyroid hormone, the nurse will instruct the patient to notify the physician if which adverse effects are noted? (select all that apply) a. palpitations b. weight gain c. angina d. fatigue e. cold intolerance

a. palpitations c. angina

Antithyroid Drugs: Propylthiouracil (PTU)

about 2 weeks of therapy may be necessary before symptoms improve, available only in oral form as 50-mg tablet, Methimazole only alternative drug and is rarely used

Interactions: Antithyroid Drugs

additive leukopenic effects when taken in conjunction with other bone marrow depressants and increase in activity of oral anticoagulants

NCLEX EXAMINATION REVIEW QUESTIONS 4. A patient with a history of renal problems is asking for advice about which antacid he should use. The nurse will make which recommendation? a. "Patients with renal problems cannot use antacids." b. "Aluminum-based antacids are the best choice for you." c. "Calcium-based antacids are the best choice for you." d. "Magnesium-based antacids are the best choice for you."

b "Aluminum-based antacids are the best choice for you."

Preparation for the NCLEX® Examination Questions 7. Which client statement demonstrates understanding of teaching by the nurse regarding the use of histamine 2 - receptor antagonists? a. "Because I am taking this medication, it is OK for me to eat spicy foods." b. "Smoking decreases the effects of this medication, so I should look into cessation programs." c. "I should take this medication 1 hour after each meal to maximally decrease gastric acidity." d. "I should increase bulk and fluids in my diet to prevent constipation."

b "Smoking decrease the effect of this medication, so I should look into cessation programs." Clients taking histamine 2 -receptor - blocking drugs should avoid spicy foods, extremes in temperatures, alcohol, and smoking. Diarrhea, not constipation, is a GI adverse effect. Whereas cimetidine should be taken with meals, famotidine can be taken without regard to meals.

Preparation for the NCLEX® Examination Questions 10. The nurse will question an order for misoprostol (Cytotec) in which patient? a. A 21-year-old man with Zollinger-Ellison syndrome b. A 32-year-old pregnant woman with a urinary tract infection c. A 45-year-old woman with GERD d. A 64-year-old man with hypertension

b A 32-year-old pregnant woman with a urinary tract infection Misoprostol (Cytotec) is a prostaglandin E analog and is believed to inhibit gastric acid secretion and protect the gastric mucosa from injury by enhancing the local production of mucus. However, it is also an abortifacient and therefore is contraindicated in pregnancy. The drug may be useful in treating patients with Zollinger-Ellison syndrome (a hypersecretory syndrome) and GERD. Hypertension is not a contraindication for its use.

NCLEX EXAMINATION REVIEW QUESTIONS 3. Which is the correct action when the nurse is administering sucralfate? a. Giving the drug with meals b. Giving the drug on an empty stomach c. Instructing the patient to restrict fluids d. Waiting 30 minutes before administering other drugs

b Giving the drug on an empty stomach

Preparation for the NCLEX® Examination Questions 6. What is the mechanism of action for Famotidine (Pepcid)? a. It forms a protective coating against gastric acid, pepsin, and bile salts. b. It competes with histamine for binding sites on the parietal cells c. It irreversibly binds to the hydrogen-potassium-adenosine triphosphate (ATPas) pump. d. It causes a decrease in stomach pH, reducing stomach acidity.

b It competes with histamine for binding sites on the parietal cells. Histamine receptor-blocking drugs decrease gastric acid by competing with histamine for binding sites on the parietal cells.

Preparation for the NCLEX® Examination Questions 8. How will the nurse describe the action of proton pump inhibitors (PPIs)? a. They form a protective barrier that can be thought of as a liquid bandage. b. They irreversibly bind to the hydrogen-potassium-ATPase pump. c. They compete with histamine for binding sites on the parietal cells d. They help to neutralize acid secretions to promote gastric mucosal defensive mechanisms.

b They irreversibly bind to the hydrogen-potassium-ATPase pump. PPIs work to block the final step in the acid-secreting mechanisms of the proton pump. They do this by irreversibly binding to the ATPase pump, H+/K+/ATPase, the enzyme for this step.

A nurse is explaining congenital adrenal hyperplasia (CAH) to a group of nursing students. Which statement by a student indicates understanding of the teaching? a. "CAH is caused by a deficiency of ACTH production." b. "CAH is the result of an inability to synthesize glucocorticoids." c. "Newborn screening provides a definitive diagnosis for CAH" d. "The enzyme 21-alpha-hydroxylase increases the production of androgens."

b. "CAH is the result of an inability to synthesize glucocorticoids." CAH results from an inborn deficiency of the enzymes needed for glucocorticoid synthesis. The condition is marked by increased production of ACTH, because the pituitary attempts to enhance glucocorticoid synthesis. Newborn screening indicates a deficiency of 21-alpha-hydroxylase; if this is noted, follow-up testing must be done to confirm a diagnosis of CAH. The enzyme 21-alpha-hydroxylase increases the production of glucocorticoids. When this enzyme is deficient, androgens are produced in excess because of stimulation by ACTH.

The nurse is providing patient education about glucocorticoid therapy to a patient preparing to be discharged home. Which statement made by the patient best demonstrates understanding of glucocorticoid therapy? a. "I will take the entire dose early with breakfast." b. "I may take two-thirds of the dose in the morning and one-third in the afternoon." c. "I will divide the dose in half and take half in the morning and half in the evening." d. "I will take a dose with each meal."

b. "I may take two-thirds of the dose in the morning and one-third in the afternoon." Patients should be instructed to follow the prescribed dosing schedule. Some prescribers recommend dividing the daily dose by taking two-thirds in the morning and one-third in the evening. Other prescribers recommend taking the entire daily dose at bedtime. Taking the dose at breakfast, dividing the dose in half, and taking a dose with each meal are not recommended dosing schedules.

A nurse is teaching a patient who will begin taking methimazole [Tapazole] for Graves' disease about the medication. Which statement by the patient indicates understanding of the teaching? a. "Because of the risk for liver toxicity, I will need frequent liver function tests." b. "I should report a sore throat or fever to my provider if either occurs." c. "I will need a complete blood count every few months." d. "It is safe to get pregnant while taking this medication."

b. "I should report a sore throat or fever to my provider if either occurs." Agranulocytosis is rare but can occur with methimazole, so patients should report signs of infection, such as a sore throat or fever. Liver toxicity is not a side effect, so liver function tests are not indicated. Because agranulocytosis often develops rapidly, periodic blood counts do not guarantee early detection. Methimazole is contraindicated in the first trimester of pregnancy.

The nurse is providing patient education to a patient who will begin taking fludrocortisone [Florinef] as adjunctive therapy to hydrocortisone. Which statement by the patient indicates understanding of the teaching? a. "I should move from sitting to standing slowly." b. "I should report any swelling of my hands and feet." c. "I should report weight loss to my provider." d. "I should report excessive urine output."

b. "I should report any swelling of my hands and feet." Fludrocortisone is a mineralocorticoid that regulates sodium, potassium, and water balance. Water and sodium retention is a particular concern, so the patient should be taught to report any signs of fluid retention, such as swelling of the hands and feet. Fludrocortisone elevates the blood pressure, so hypotension is not a concern. Because fluid retention causes weight gain, patients should be taught to report any increase in weight. Patients should report decreased urine output.

The nurse prepares a patient with Graves' disease for radioactive iodine (131I) therapy. Which statement made by the patient best demonstrates understanding of 131I therapy? a. "I will have to isolate myself from my family for 1 week so that I don't expose them to radiation." b. "This drug will be taken up by the thyroid gland and will destroy the cells to reduce my hyperthyroidism." c. "This drug will help reduce my cold intolerance and weight gain." d. "I will need to take this drug on a daily basis for at least 1 year."

b. "This drug will be taken up by the thyroid gland and will destroy the cells to reduce my hyperthyroidism." Iodine-131 can be used to destroy thyroid tissue in patients with hyperthyroidism; no further teaching is necessary. The patient does not need to isolate himself from others. The treatment will not reduce intolerance to cold, nor will it affect weight gain. The patient will not need the treatment daily.

A nurse caring for a patient notes that the patient has a temperature of 104°F and a heart rate of 110 beats per minute. The patient's skin is warm and moist, and the patient complains that the room is too warm. The patient appears nervous and has protuberant eyes. The nurse will contact the provider to discuss: a. cretinism. b. Graves' disease. c. myxedema. d. Plummer's disease.

b. Graves' disease. The signs and symptoms in this patient are consistent with hyperthyroidism, and because the patient's eyes are protuberant, they also are consistent with Graves' disease. Cretinism is hypothyroidism in children. Myxedema is severe hypothyroidism. Plummer's disease is a hyperthyroidism condition without exophthalmos.

The nurse is caring for a pregnant patient recently diagnosed with hypothyroidism. The patient tells the nurse she does not want to take medications while she is pregnant. What will the nurse explain to this patient? a. Hypothyroidism is a normal effect of pregnancy and usually is of no consequence. b. Neuropsychologic deficits in the fetus can occur if the condition is not treated. c. No danger to the fetus exists until the third trimester. d. Treatment is required only if the patient is experiencing symptoms.

b. Neuropsychologic deficits in the fetus can occur if the condition is not treated. Maternal hypothyroidism can result in permanent neuropsychologic deficits in the child. Hypothyroidism is not a normal effect of pregnancy and is a serious condition that can affect both mother and fetus. The greatest danger to the fetus occurs in the first trimester, because the thyroid does not fully develop until the second trimester. Early identification is essential. Symptoms often are vague. Treatment should begin as soon as possible, or mental retardation and other developmental problems may occur.

The nurse is teaching a patient who has a new prescription for the antithyroid drug propylthiouracil (PTU). Which statement by the nurse is correct? a. There is no food restrictions b. You need to avoid foods high in iodine, such as iodized salt, seafood, and soy products c. This drug is given to raise the thyroid hormone levels in your blood d. Take this drug in the morning on an empty stomach

b. You need to avoid foods high in iodine, such as iodized salt, seafood, and soy products

When monitoring the laboratory values of a patient who is taking antithyroid drugs, the nurse knows to watch for a. increased platelet counts b. decreased WBC counts c. decreased BUN level d. increased blood glucose levels

b. decreased WBC counts

A patient is admitted to the hospital and will begin taking levothyroxine [Synthroid]. The nurse learns that the patient also takes warfarin [Coumadin]. The nurse will notify the provider to discuss ____ the ____ dose. a. reducing; levothyroxine b. reducing; warfarin c. increasing; levothyroxine d. increasing; warfarin

b. reducing; warfarin Levothyroxine accelerates the degradation of vitamin K-dependent clotting factors, which enhances the effects of warfarin. Patients taking warfarin who start taking levothyroxine may need to have their warfarin dose reduced. It is not correct to increase or decrease the levothyroxine dose or to increase the warfarin dose.

Preparation for the NCLEX® Examination Questions 9. PPIs have the ability to almost totally inhibit gastric acid secretion. Because of this possibility, the use of the medication can lead to what condition? a. Gastric ulcer formation b. Gastroesoophageal reflux disease (GERD) c. Achlohydria d. Diverticulosis

c Achlohydria Becasue PPIs stop the final step of acid secretion, they can block up to 90% of acid secretion, leading to achlorhydria (without acid).

Preparation for the NCLEX® Examination Questions 12. How dose sucralfate (Carafate) achieve a therapeutic effect? a. By inhibiting the production of gastric acid secretion b. by enhancing gastric absorption c. By forming a protective barrier over the gastric mucosa d. By neutralizing gastric acid

c By forming a protective barrier over the gastric mucosa Sucralfate has a local effect only on the gastric mucosa. If forms a protective barrier that can be thought of as a liquid protective bandage in the stomach. This liquid bandage adheres to the gastric lining, protecting against adverse effects related to gastric acid. It also stimulates healing of any ulcerated areas of the gastric mucosa.

Preparation for the NCLEX® Examination Questions 11. The nurse would teach a client prescribed simethicone (Mylicon) to avoid which substance? a. Over-the-counter antacids b. Histamin 2- receptor antagonists c. Carbonated beverages d. Milk and dairy products

c Carbonated beverage Simethicone is used to decrease gas and belching. both of which can be aggravated or caused by ingesting carbonated beverages. It may be given in combination with other medications used to decrease acidity.

NCLEX EXAMINATION REVIEW QUESTIONS 2. When evaluating the medication list of a patient who will be starting therapy with an H2 receptor antagonist, the nurse is aware that which drug may interact with it? a. codeine b. pencillin c. Phenytoin d. acetaminophen

c Phenytoin

NCLEX EXAMINATION REVIEW QUESTIONS 8. The nurse is preparing to administer the first dose of misoprostol (Cytotec) for a patient who has been diagnosed with a gastric ulcer. What condition would be a contraindication to this medication? a. Hypothyroidism b. Type 2 diabetes mellitus c. Pregnancy d. Hypertension

c Pregnancy

A patient is given 1 mg of dexamethasone at 11:00 PM; a plasma cortisol level recorded at 8:00 PM the next day is normal. The nurse knows that this is an indication that the patient has what condition? a. Addison's disease b. Congenital adrenal hyperplasia c. Cushing's syndrome d. Secondary adrenal insufficiency

c. Cushing's syndrome The overnight dexamethasone suppression test, which is performed by administering dexamethasone as described, is used to diagnose Cushing's syndrome. In normal individuals, dexamethasone suppresses the release of adrenocorticotropic hormone (ACTH), thereby suppressing the synthesis and release of cortisol, which results in a low cortisol level. In patients with Cushing's syndrome, the cortisol level is normal or only mildly low. This is not an indication of Addison's disease, which is diagnosed by administering cosyntropin. Neither test is useful for diagnosing congenital adrenal hyperplasia or secondary adrenal insufficiency.

A nurse obtaining an admission history on an adult patient notes that the patient has a heart rate of 62 beats per minute, a blood pressure of 105/62 mm Hg, and a temperature of 96.2°F. The patient appears pale and complains of always feeling cold and tired. The nurse will contact the provider to discuss tests for which condition? a. Cretinism b. Graves' disease c. Hypothyroidism d. Plummer's disease

c. Hypothyroidism This patient is showing signs of hypothyroidism: a low heart rate, low temperature, pale skin, and feeling cold and tired. In adults, thyroid deficiency is called hypothyroidism. In children, thyroid deficiency is called cretinism. Graves' disease and Plummer's disease are conditions caused by thyroid excess.

A patient with hypothyroidism begins taking PO levothyroxine [Synthroid]. The nurse assesses the patient at the beginning of the shift and notes a heart rate of 62 beats per minute and a temperature of 97.2°F. The patient is lethargic and difficult to arouse. The nurse will contact the provider to request an order for which drug? a. Beta blocker b. Increased dose of PO levothyroxine c. Intravenous levothyroxine d. Methimazole [Tapazole]

c. Intravenous levothyroxine Intravenous administration of levothyroxine is used for myxedema coma. This patient is showing signs of severe hypothyroidism, or myxedema. A beta blocker is useful in patients who show signs of hyperthyroidism to minimize cardiac effects. Because the half-life of oral levothyroxine is so long, increasing the PO dose will not provide immediate relief of this patient's symptoms. Methimazole is used to treat hyperthyroidism.

A patient in her twenties with Graves' disease who takes methimazole [Tapazole] tells a nurse that she is trying to conceive and asks about disease management during pregnancy. What will the nurse tell her? a. Methimazole is safe to take throughout pregnancy. b. Propylthiouracil should be taken throughout her pregnancy. c. The patient should discuss changing to propylthiouracil from now until her second trimester with her provider. d. The patient should discuss therapy with iodine-131 instead of medications with her provider.

c. The patient should discuss changing to propylthiouracil from now until her second trimester with her provider. Methimazole is not safe during the first trimester of pregnancy, because it is associated with neonatal hypothyroidism, goiter, and cretinism; however, it is safe in the second and third trimesters. Propylthiouracil is recommended for pregnant patients only in the first trimester and during lactation only if a thionamide is absolutely necessary. Iodine-131 is used in women older than 30 years who have not responded to medication therapy and is contraindicated during pregnancy

A patient who has chronic adrenal insufficiency is admitted to the hospital for an open cholecystectomy. The nurse obtaining the admission history learns that the patient takes hydrocortisone 25 mg PO daily in the morning. The patient's surgery is scheduled for the next morning. The nurse will expect an order to: a. administer the usual morning dose of hydrocortisone 25 mg PO. b. administer hydrocortisone 50 mg PO in the morning. c. administer hydrocortisone 50 mg IV before surgery. d. withhold the morning dose of hydrocortisone and give it after surgery.

c. administer hydrocortisone 50 mg IV before surgery. Patients who take steroids need extra steroid before situations that cause stress, such as surgery. Failure to administer the increased dose can prove fatal. For surgeries that cause moderate stress, such as a cholecystectomy, patients should be given 50 mg of hydrocortisone intravenously the day of the procedure, followed by a taper over 1 to 2 days to the usual replacement dose. Giving the usual dose or giving an increased oral dose is not indicated

To help with the insomnia associated with thyroid hormone replacement therapy, the nurse will teach the patient to a. take half the dose at lunchtime and the other half 2 hours later b. use a sedative to assist with falling asleep c. take the dose upon awakening in the morning d. reduce the dosage as needed if sleep is impaired

c. take the dose upon awakening in the morning

NCLEX EXAMINATION REVIEW QUESTIONS 1. A 30-year-old patient is taking simethicone for excessive flatus associated with diverticulitis. During a patient teaching session, the nurse explains the mechanism of action of simethicone by saying: a. "It neutralize gastric pH, thereby preventing gas." b. "It buffers the effects of pepsin on the gastric wall." c. "It decreases gasgtric acid secretion and thereby minimizes flatus." d. "It causes mucus-coated gas bubbles to break into smaller ones."

d "It causes mucus-coated gas bubbles to break into smaller ones."

Preparation for the NCLEX® Examination Questions 3. For a client with chronic renal failure, the nurse MOST likely will question a prescription for which type of antacid? a. Aluminum-containing antacids b. Calcium-containing antacids c. Sodium-containing antacids d. Magnesium-containing antacids

d Magnesium-containing antacids Magnesium-containing antacids can cause hypermagnesemia in clients with chronic renal failure. Aluminum-containing antacids may be used as a phosphate binder in clients with chronic renal failure. Sodium-and aluminum-containing antacids are chemically more easily excreted in clients with renal compromise. although calcium-containing antacids may accumulate in the bloodstream of clients with renal failure, they may also be appropriate because these patients may be hypocalcemic.

Preparation for the NCLEX® Examination Questions 4. What condition will the nurse monitor for with a client using sodium bicarbonate to treat gastric hyperacidity? a. Hypercalcemia b. Hyperkalemia c. Metabolic acidosis d. Metabolic alkalosis

d Metabolic alkalosis Solutions containing sodium bicarbonate (a base) can cause metabolic alkalosis. Serum potassium and serum calcium would decrease, not increase, with alkalosis.

Preparation for the NCLEX® Examination Questions 5. Which nursing diagnosis is appropriate for a patient receiving famotidine (Pepcid)? a. Ineffective peripheral tissue perfusion related to hypertension b. Risk for infection related to immunosuppression c. Impaired urinary elimination related to retention d. Risk for injury related to thrombocytopenia

d Risk for injury related to thrombocytopenia. A serious side effect of famotidine is thrombocytopenia, which is manifested by a decrease in platelet count and an increased risk of bleeding. The patient receiving famotidine may experience hypotension as an adverse effect, not hypertension. Famotidine dose not cause immunosuppression or urinary retention.

A nurse is teaching a patient who has been diagnosed with hypothyroidism about levothyroxine [Synthroid]. Which statement by the patient indicates a need for further teaching? a. "I should not take heartburn medication without consulting my provider." b. "I should report insomnia, tremors, and an increased heart rate to my provider." c. "If I take a multivitamin with iron, I should take it 4 hours after the Synthroid." d. "If I take calcium supplements, I may need to decrease my dose of Synthroid."

d. "If I take calcium supplements, I may need to decrease my dose of Synthroid." Patients taking calcium supplements should take these either 4 hours before or after taking levothyroxine, because they interfere with levothyroxine absorption. Many heartburn medications contain calcium, so patients should consult their provider before taking them. Insomnia, tremors, and tachycardia are signs of levothyroxine toxicity and should be reported. Iron also interferes with levothyroxine absorption, so dosing should be 4 hours apart.

A patient has a free T4 level of 0.6 ng/dL and a free T3 of 220 pg/dL. The patient asks the nurse what these laboratory values mean. How will the nurse respond? a. "These laboratory values indicate that you may have Graves' disease." b. "These results suggest you may have hyperthyroidism." c. "We will need to obtain a total T4 and a total T3 to tell for sure." d. "We will need to obtain a TSH level to better evaluate your diagnosis."

d. "We will need to obtain a TSH level to better evaluate your diagnosis." A free T4 level of less than 0.9 ng/dL and a free T3 of less than 230 pg/dL are consistent with hypothyroidism, but measurement of the thyroid-stimulating hormone (TSH) level is necessary to distinguish primary hypothyroidism from secondary hypothyroidism. Total T3 and T4 levels are not as helpful as free T3 and T4 levels. These laboratory values indicate hypothyroidism, not hyperthyroid conditions such as Graves' disease.

A patient who has been newly diagnosed with adrenal hormone deficiency will begin taking hydrocortisone. The nurse provides teaching for this patient. Which statement by the patient indicates understanding of the teaching? a. "I may take all of my daily dose in the morning or divide it in half and take it twice daily." b. "I will need to take this medication until my symptoms completely clear, and then I may stop." c. "Side effects are common with hydrocortisone, even with therapeutic doses." d. "When I am sick, I should take three times the normal dose for 3 days in a row."

d. "When I am sick, I should take three times the normal dose for 3 days in a row." Patients who take hydrocortisone as replacement need to be taught to increase their daily dose during times of stress, because the drug causes adrenal suppression, and the adrenals will not be able to release cortisone as usual during stress. A general rule of thumb is the "3 by 3 rule": take 3 times the usual dose for 3 days when sick. The daily dose is usually given once in the morning; if late-day fatigue occurs, patients may split the dose and take two-thirds in the morning and one-third in the late afternoon or evening. Treatment is lifetime. Side effects are uncommon when hydrocortisone is given in therapeutic doses.

A patient with Cushing's syndrome has undergone surgery and radiation treatment. The nurse will expect to teach the patient about which medication? a. Cosyntropin b. Dexamethasone c. Fludrocortisone acetate [Florinef] d. Ketoconazole [Nizoral]

d. Ketoconazole [Nizoral] The role of drugs in the treatment of Cushing's syndrome is limited; drugs are used only as adjuncts to surgery and radiation therapy. The most effective agent is ketoconazole, which suppresses steroid synthesis. Cosyntropin is used to diagnose adrenal insufficiency. Dexamethasone is a glucocorticoid used for replacement therapy and to diagnose Cushing's syndrome. Fludrocortisone is used for chronic mineralocorticoid replacement.

The pharmacy has called a patient to notify her that the current brand of thyroid replacement hormone is on back order. The patient calls the clinic to ask what to do. Which is the best response by the nurse? a. Go ahead and take the other brand that the pharmacy has available for now b. You can stop the medication until your current brand is available c. You can split the thyroid pills that you have left so that they will last longer d. Let me ask your physician what needs to be done; we will need to watch how you do if you switch brands

d. Let me ask your physician what needs to be done; we will need to watch how you do if you switch brands

A patient who has been taking 25 mg of hydrocortisone each morning for several months reports feeling fatigued late in the day each day. What will the nurse tell the patient to discuss with the provider? a. Adding a mineralocorticoid to the drug regimen b. Assessing serum electrolytes to check for toxicity c. Increasing the dose to 50 mg daily d. Splitting the daily dose into a morning and an afternoon dose

d. Splitting the daily dose into a morning and an afternoon dose Patients generally take hydrocortisone once daily in the morning to mimic the body's natural cortisol release cycle. However, some patients develop fatigue late in the day. These patients may split the daily dose, taking two-thirds in the morning and one-third in the afternoon or early evening. Mineralocorticoids are given to maintain intravascular volume and regulate sodium, potassium, and hydrogen, so adding this drug would not help with fatigue. Fatigue is not a sign of steroid toxicity. Increasing the dose is not indicated.

An older adult patient is diagnosed with hypothyroidism. The initial free T4 level is 0.5 mg/dL, and the TSH level is 8 microunits/mL. The prescriber orders levothyroxine [Levothroid] 100 mcg/day PO. What will the nurse do? a. Administer the medication as ordered. b. Contact the provider to discuss giving the levothyroxine IV. c. Request an order to give desiccated thyroid (Armour Thyroid). d. Suggest that the provider lower the dose.

d. Suggest that the provider lower the dose. In older adult patients, initial dosing of levothyroxine should start low and be increased gradually. A typical starting dose for an elderly patient is 25 to 50 mcg/day. It is not correct to administer the medication without questioning the provider. Unless the patient has signs of myxedema, there is no need to give the medication IV. Desiccated thyroid is no longer used except in patients who have been taking it long term.

When assessing the elderly patient, the nurse keeps in mind that certain nonspecific symptoms may represent hypothyroidism in these patients, such as: a. leukopenia, anemia b. loss of appetite, polyuria c. weight loss, dry cough d. cold intolerance, depression

d. cold intolerance, depression

A patient arrives in the emergency department with a heart rate of 128 beats per minute and a temperature of 105°F. The patient's skin feels hot and moist. The free T4 level is 4 ng/dL, the free T3 level is 685 pg/dL, and the TSH level is 0.1 microunits/mL. The nurse caring for this patient will expect to administer: a. intravenous levothyroxine. b. iodine-131 (131I). c. methimazole [Tapazole]. d. propylthiouracil (PTU).

d. propylthiouracil (PTU). Propylthiouracil is used for patients experiencing thyroid storm, and this patient is showing signs of that condition. Levothyroxine is given IV for hypothyroidism. 131I is used in patients over age 30 who have not responded to other therapies. Methimazole is used long term to treat hyperthyroidism, but PTU is more useful for emergency treatment.

Factors to be considered before initiation of drug therapy with thyroid drug

desire ratio T3 to T4, the cost, and the desired duration of effect

Mechanism of Action: Antithyroid Drugs: Methimazole and Propylthiouracil

inhibit the incorporation of iodine molecules into the amino acid tyrosine, a process required to make the precursors of T3 and T4- impede formation of thyroid hormone **propylthiouracil has added effect of inhibiting conversion of T4 to T3 in peripheral circulation **neither can inactivate existing thyroid hormone -effects primarily limited to thyroid gland and overall effect is to decrease thyroid hormone level

Contraindications: Thyroid drugs

known drug allergy, recent myocardial infarction, adrenal insufficiency, and hyperthyroidism

Contraindications: Antithyroid Drugs

known drug allergy, use in pregnancy is controversial-according to FDA propythiouracil to be used during first trimester only, then methimazole is used for remainder f pregnancy *case reports of scalp abnormalities when methimazole is used, both drugs classified as pregnancy D drugs

Adverse Effects: Antithyroid Drugs

most serious are liver and bone marrow toxicity, drowsiness, vertigo, paresthesia, hepatitis, loss of taste, smoky urine, decreased urine output, agranulocytosis, leukopenia, thrombocytopenia, hypothrominemia, lymphadenopathy,bleeding, rash, pruritus, myalgia, arthralgia, increased BUN and serum creatinine levels, enlarged thyroid gland nephritis

Contraindications: Thyroid Drugs

pregnancy A drugs, hypersensitivity, adrenal insufficiency, previous myocardial infarction, or hyperthyroidism

Indications: Thyroid Drugs

replace what the thyroid gland itself cannot produce to achieve normal thyroid hormone levels (euthyroid condition), also used for diagnoses of suspected hyperthyroidism (TSH-suppression test) and prevent or treat various types of goiters, used for replacement hormonal therapy in patients whose thyroid gland was removed or destroyed during radiation for cancer or hyperthyroidism

Indications: Antithyroid Drugs

treat hyperthyroidism and prevent the surge in thyroid hormones that occurs after the surgical treatment of or during radioactive iodine therapy for hyperthyroidism or thyroid cancer **long-term use of these drugs during treatment of hyperthyroidism like during Graves' disease (several years) may induce spontaneous remission

Hyperthyroidism during Pregnancy

treated with dosage adjustments every 4 weeks to maintain TSH level at the lower end of normal range- fetal growth may be retarded if maternal hypothyroidism remains untreated during pregnancy

Adverse Effects: Thyroid drugs

usually due to overdose; most significant is cardiac dysrhythmia with the risk of life-threatening or fatal irregularities, tachycardia, palpitations, angina, hypertension, insomnia, tremors, anxiety, menstrual irregularities, weight loss, sweating, heart intolerance, fever

Mechanism of Action: Thyroid Drugs

work in same manner as endogenous thyroid hormones, affecting many body systems; cellular level-induce the metabolic rate like the rate of protein, carbohydrate, and lipid metabolism, increase oxygen consumption, body temperature, blood volume, and overall cellular growth and differentiation. stimulate cardiovascular system by increasing number of myocardial beta-adrenergic receptors - increases sensitivity of heart to catecholamines and increases cardiac output. also increase renal blood flow and glomerular filtration rate that results in diuretic effect


Set pelajaran terkait

History - Chapter 5 (Sections 4-?)

View Set

Fundamental Accounting Principles, 24e Study

View Set

PSYC325 Final Exam: Lectures 17 and 18

View Set

Chapter 5 Constrained optimization and consumption

View Set